Sie sind auf Seite 1von 51

CHAPTER

04
Moving Charges
and Magnetism
In this chapter, we will be going to study magnetism produced by a moving
charge and further we will proceed with Ampere’s circuital law and its
applications and at last, the chapter will be ended with magnetic force and
torque between two parallel conductors. All the topics mentioned above are
discussed in detail, so it will be more interesting to understand them very
carefully after going through each and every sentence very thoroughly.

TOPIC Magnetic Field and


01 Its Applications Magnetic Field and Its
Applications
Ampere’s Circuital Law
The space in the surroundings of a magnet or a current carrying conductor and Moving Charges
in which its magnetic influence can be experienced is called magnetic field. Magnetic Force and Torque
The SI unit of magnetic field is tesla (T) or weber/metre 2 (Wbm −2 ) or Experienced by a Current Loop
NA −1m −1 and its CGS unit is gauss (G).
1 tesla = 10 4 gauss

Oersted’s Experiment
Oersted experimentally demonstrated that the current carrying conductor
produces magnetic field around it. His experimental arrangement is shown
below. A conducting wire AB is connected to the poles of a battery through
a key ( K ). When key K is closed, deflection occurs in compass needle and
vice-versa.
Battery
K

I (Current)
A B
Conducting wire

Compass needle
164 |

In vector form, Biot-Savart’s law can be written as


Magnetic Field due to
Id l × r µ 0 Id l × r
a Current Element : dB∝ 3
=

⋅ 3 ...(i)
r r
Biot-Savart’s Law From Eq. (i), the direction of d B would be the direction
Biot-Savart’s law is an experimental law predicted by Biot of the cross-product vector ( d l × r), which is represented
and Savart. This law deals with the magnetic field by the right handed screw rule or right hand thumb rule.
induction at a point due to a small current element (a Here, d B is perpendicular to the plane containing d l
part of any conductor, carrying current).
and r is directed inwards (since, point P is to the right of
Let XY be current carrying conductor, I be current in the current element).
the conductor, dl be infinitesimal small element of the Magnetic field induction at point P due to current
conductor, dB be magnetic field at point P at a distance r through entire wire is
from the element.
µ Id l × r
Y
B= ∫ 0 ⋅ 3
Current 4π r
element µ Id l sin θ
Idl θ or B=∫ 0⋅
4π r2
r
⊗ dB Biot-Savart’s law in terms of current density J can be
P
I
written as
µ J×r  I Idl Idl 
d B = 0 ⋅ 3 dV Q J = = =
4π r  A Adl dV 
X where, J = current density at any point on the current
According to Biot-Savart’s law, the magnitude of element and dV = volume of the element.
magnetic field induction ( dB ) at a point P due to a Biot-Savart’s law in terms of charge (q) and its velocity (v )
current element depends on the following factors can be written as
(i) dB ∝ I (i.e. magnetic field is directly proportional µ q( v × r)  q dl 
to the current flowing through the conductor). dB = 0 ⋅ 3 Q Id l = d l = q = q v 
4π r  dt dt 
(ii) dB ∝ dl (i.e. magnetic field is directly proportional
Biot-Savart’s law in terms of magnetising force or
to the length of the element).
magnetising intensity (H ) of the magnetic field is
(iii) dB ∝ sin θ (i.e. magnetic field is directly proportional in SI or MKS system,
to the sine of angle between the length of element
and line joining the element to point (P).
dB 1 Id l × r 1 Id l × r$
dH = = ⋅ 3 = ⋅
1 µ 0 4π r 4π r2
(iv) dB ∝ 2 (i.e. magnetic field is inversely
r 1 Idl sin θ
∴ dH = ⋅
proportional to the square of distance between the 4π r2
element and point P). Combining all the above Id l × r Idl sin θ
Idl sin θ In CGS units, d H = 3
and dH =
relations, dB ∝ r r2
r2
This relation is called Biot-Savart’s law. Permittivity and Permeability
If conductor is placed in air or vacuum, then magnetic The relation between ε 0 and µ 0 is always a constant, i.e.
field is given by −7
 µ  1 × 10 1 1
µ Idl sin θ µ 0 ε 0 = (4 πε 0 ) ×  0  = = = 2
dB = 0 ⋅  4 π  9 × 10 9 8
(3 × 10 ) 2
c
4π r2
1
µ ⇒ c=
where, 0 is a proportionality constant, µ 0 is the µ 0ε 0

permeability of free space. µ 0 = 4 π × 10 −7 Tm /A where, µ 0 = 4 π × 10 −7 T /m
(or Wb/A-m), its dimensions are [MLT −2 A −2 ] . and ε 0 = 8.86 × −12 Nm 2C −2
Moving Charges and Magnetism | 165

Magnetic Field due to Maxwell’s Cork Screw Rule


a Current Carrying Conductor According to this rule, if we imagine a right handed cork
Let us consider a straight conductor XY , carrying a screw placed along the current carrying wire conductor,
current I and P is a point at which the magnetic field is rotated such that the screw moves in the direction of
required. current, then the direction of rotation of the screw gives
the direction of magnetic field lines.
Y φ
dφ Direction of
F G φ2
dl magnetic
E field
P Direction
φ1
of current

X Example 1 A current 10 A is flowing East to West in


µ I a long wire kept horizontally in the East-West direction.
B = 0 (sin φ1 + sin φ 2 )
4 πr Find the magnitude and direction of magnetic field in a
According to right hand thumb rule, the direction of horizontal plane at a distance of 10 cm North.
magnetic field in this case is perpendicular to the plane of Sol. Given, current, I = 10 A (East to West)
paper and directed inwards. Distance, r = 10 cm = 10 × 10 − 2 m
Magnetic field B for infinitely long wire, at any points Magnetic field, | B | = ?
except end point of wire.
10 cm
As, φ1 = φ 2 = π /2
West East
10 A
µ 0I
then B1 = The magnitude of magnetic field |B| for infinite length of
2 πr µ I 4 π × 10−7 × 10
wire = 0 ⇒ | B | = = 2 × 10− 5 T
Magnetic field for infinitely long wire, near the end 2π r 2 × π × 10 × 10− 2
points of wire, φ1 = 0° and φ 2 = 90° The direction of magnetic field is given by right hand thumb
rule or Maxwell’s cork screw rule. So, the direction of magnetic
µ0 I B
B2 = ⋅ ⇒ Clearly, B 2 = 1 field at point 10 cm North due to flowing current is
4π r 2 perpendicularly inwards to the plane of paper.
The direction of the magnetic field associated with a Example 2 Find an expression for the magnetic field
current carrying conductor can be determined by right
hand thumb rule or Maxwell’s cork screw rule. at the centre of a coil bent in the form of square of side
2a, carrying current I as shown in the figure.
Right Hand Thumb Rule Sol. Given, θ1 = 45°, θ 2 = 45°
According to this rule, if we imagine a linear wire 2a
A B
conductor to be held in the grip of the right hand such
that the thumb points in the direction of current, then a
45° 2a
O
the curvature of the fingers around the conductor will a I 45°
give the direction of magnetic field lines.
D C
2a
Total magnetic field due to each side at point O is given by
µ I
Direction of B = 4 0 (sin θ1 + sin θ 2 )
magnetic 4 πa
Direction
field µ 0I
of current =4 (sin 45 °+ sin 45 ° )
4 πa
µ I 1 1  2 µ 0I
= 0  +  =
πa  2 2 πa
166 |

Magnetic Field on the Axis of a Circular µ0 Ia


4 π (r 2 + a 2 )3 / 2 ∫
= dl
Current Carrying Loop
Let us consider a circular loop of radius a with centre C. µ Ia
= 0 2 (2 πa )
Let the plane of the coil be perpendicular to the plane of 4 π (r + a 2 )3 / 2
the paper and current I be flowing in the direction
shown. Suppose P is any point on the axis of a coil at a µ 0 Ia 2
distance r from the centre C. or B= … (ii)
2(r 2 + a 2 )3 / 2
L dl
dB cos φ

Q
√r 2 + a 2 dB For N turns, the net magnetic field is given by
a φ
φ P µ 0 NIa 2
I C dB sin φ B=
r φ dB sin φ 2 (r 2 + a 2 )3 / 2
I φ
dB cos φ At the centre, r = 0
dB
M
dl
Q′ µ 0 NIa 2
B=
Now, consider a current element Idl on top ( L), where 2(0 + a 2 )3 / 2
current comes out of paper normally, whereas at bottom µ NI
( M ), current enters into the plane paper normally. B= 0
2a
Q LP ⊥ dl The direction of B is along the axis and away from the
Also, MP ⊥ dl loop, when current in the coil is in anti-clockwise
direction.
∴ LP = MP = r 2 + a 2
B
µ0 NI
The magnetic field at point P due to the current
2a
element Idl, according to Biot-Savart’s law is given by
µ Idl sin 90° µ 0 . Idl
dB = 0 ⋅ 2 = X
O
r
4 π (r + a 2 ) 4 π (r 2 + a 2 )
where, a = radius of circular loop Y
and r = distance of point P from the centre C Variation of magnetic field
induction (B) with distance r
along the axis.
According to right hand screw rule, the direction of dB is Example 3 A circular coil of 120 turns has a radius
perpendicular to LP and along PQ, where PQ ⊥ LP .
of 18 cm and carries a current of 3 A. What is the
Similarly, the same magnitude of magnetic field is
magnitude of the magnetic field at a point on the axis
obtained due to current element Idl at the bottom and of the coil at a distance from the centre equal to the
direction is along PQ ′, where PQ ′ ⊥ MP . radius of the circular coil?
Now, resolving dB due to current element at L and Sol. Given, number of turns N = 120, current I = 3A,
M . So, dB cos φ components balance each other and net radius of coil, r = 18 cm = 0.18 m and
magnetic field is given by distance from the centre to a point on axis, a = r = 0.18 m
µ0  Idl  a µ 0 NI a 2
As, B =
B = ∫ dB sin φ = ∫ 4 π  r 2 + a 2  ⋅ 2(a 2 + r 2 )3 / 2
r + a2
2
4 π × 10− 7 × 120 × 3 × (0.18)2
  =
QIn ∆PCL,sin φ =
a
 2[(0.18)2 + (0.18)2 ]3 /2
 r + a2
2  ⇒ B = 4.4 × 10 − 4 T
 
Moving Charges and Magnetism | 167

Magnetic Field at the Centre of a Example 4 A circular coil of wire consisting of


Current Carrying Circular Loop 100 turns, each of radius 8.0 cm carries a current of 0.40
Consider a circular loop of radius R carrying current I. A. What is the magnitude of the magnetic field B at the
centre of the coil? NCERT
−2
Sol. Here, n = 100, r = 8cm = 8 × 10 m and I = 040A
.
R
C ∴ Magnetic field B at the centre,
I µ 2 πIn 10− 7 × 2 × 3.14 × 0.40 × 100
B= 0⋅ = = 3 .1 × 10 −4 T
4π r 8 × 10−2
Magnetic field at its centre C is given by
Example 5 The magnetic field B due to a current
µ I
B= 0 carrying circular loop of radius 12cm at its centre is
2R
0.5 × 10 –4 T. Find the magnetic field due to this loop at a
is obtained by setting r = 0 in previous relation (ii). point on the axis at a distance of 5.0 cm from the
If we take a coil having N number of turns, then centre.
magnetic field at its centre is Sol. Magnetic field at the centre of a circular loop,
µ 0I
µ NI B1 =
B= 0 2R
2R µ 0 IR 2
and that at an axial point, B2 =
The direction of magnetic field at the centre of circular 2 ( R 2 + x 2 )3 /2
loop is given by right hand rule. B2 R3  R3 
Thus, = 2 or B 2 = B 1  2 3 /2 
Similarly, magnetic field at the centre of a semi-circular B1 ( R + x ) 2 3 /2 2
( R + x ) 
µ I
wire of radius R, carrying current I is given by , B = 0 . Substituting the values, we have
4R  ( 12 ) 3 
B 2 = (0.5 × 10–4 )  3 /2 
= 3.9 × 10–5 T
Right Hand Rule  ( 144 + 25 ) 
According to this rule, if we hold the thumb of right Example 6
hand mutually perpendicular to the grip of fingers such An electric current is flowing in a
that the curvature of fingers depicts the direction of circular coil of radius a. At what distance from the
current in circular wire loop, then the thumb will point centre on the axis of the coil will the magnetic field be
1
in the direction of magnetic field near the centre of loop. th of its value at the centre?
8
Sol. Magnetic field induction at a point on the axis at
distance x from the centre of the circular coil carrying
current is
µ 2πnIa 2
B1 = 0 ⋅ 2
4 π ( a + x 2 )3 /2
Magnetic field induction at the centre of the circular coil
carrying current is
µ 2 πnI
B2 = 0 ⋅
Note As current carrying loop has the magnetic field lines around 4π a
it thus, it behaves as a magnet with two mutually opposite poles. B
But as per question, B1 = 2
8
µ0 2 πnIa 2 µ 0 2 πnI 1
⇒ ⋅ = ⋅ ×
4 π ( a 2 + x 2 )3 / 2 4 π a 8
a2 1
⇒ 2 2 3 /2
= ⇒ 8 a3 = ( a 2 + x 2 )3 / 2
(a + x ) 8 a
The anti-clockwise flow of current behaves like a North pole, ⇒ 2 a = ( a 2 + x 2 )1 / 2
whereas clockwise flow as South pole. ⇒ 4a 2 = a 2 + x 2 ⇒ x = 3a
168 |

Example 7 A long insulated copper wire is closely 3. There is a thin conducting wire carrying
wound as a spiral of N turns. The spiral has inner radius current. What is the value of magnetic field
a and outer radius b. The spiral lies in the XY-plane and induction at any point on the conductor itself?
a steady current I flows through the wire. Find the (a) 1 (b) Zero
Z-component of the magnetic field at the centre of the (c) − 1 (d) Either (a) or (b)
spiral.
4. A straight wire carrying a current of 12 A is sent
Y
into a semicircular loop of radius 2.0 cm. What
will be magnetic field at the centre of the
I a
semicircular loop?
b X

I I

Sol. If we take a small strip of dr at distance r from centre, (a) B = 1.9 × 10− 4 T
then number of turns in this strip would be (b) B = 1.9 × 104 T
 N  (c) B = 1.9 × 10− 4 T, ⊗
dN =   dr
b − a (d) B = 1.9 × 10 4 T, ⊗
Magnetic field due to this element at the centre of the coil will 5. An infinitely long conductor PQR is bent to
be
µ ( dN )I µ 0 NI dr form a right angle as shown. A current I flows
dB = 0 = through PQR. The magnetic field due to this
2r 2 (b − a ) r
r =b
current at the point M is H 1 . Now, another
∴ B= ∫r = a dB infinitely long straight conductor QS is
µ 0 NI connected at Q, so that the current is 1/2 in QR
b
= ln   as well as in QS. The current in PQ remaining
2 (b − a )  a 
unchanged. The magnetic field at M is now H 2.
The ratio H 1 / H 2 is given by

Topic Practice 1 M

Multiple Choice Questions –¥


90°

P I Q S
1 Mark
R –¥
1. Choose the correct option of : The Biot-Savart’s
1
law in vector form is ISC 2013 (a) (b) 1
µ dl ( I × r ) µ I (dl × r ) 2
(a) d B = 0 (b) d B = 0 2
4π r3 4π r3 (c) (d) 2
µ I ( r × dl ) µ I (dl × r ) 3
(c) d B = 0 (d) d B = 0
4π r3 4π r2
Very Short Answer Type Questions
2. An element ∆I = ∆x $i is placed at the origin and
1 Mark
carries a current I = 10 A.
Y 6. In what respect does a wire carrying a current
differ from a wire, which carries no current?
P
7. How can you justify that a current carrying wire
0.5 m produces magnetic field?

X
8. Give the dependence of magnetic field
∆x produced by a current conductor.

If ∆x = 1 cm, magnetic field at point P is 9. State Biot-Savart’s law and express this law in
(a) 4 × 10 k$ T
−8
(b) 4 × 10 $i T
−8 the vector form.

(c) 4 × 10− 8 $j T (d) − 4 × 10− 8 $j T 10. Among Biot-Savart’s law and Coulomb’s law,
which one is angle dependent?
Moving Charges and Magnetism | 169

11. Figure below shows a point P near a long 18. An element ∆l = ∆x $i is placed Y
conductor XY carrying a current I. at the origin (as shown in P
MN is a short current conductor, kept at the figure) and carries a current
point P, parallel to the conductor XY. I = 2A. Find out the magnetic
What is the direction of magnetic flux density B field at a point P on the O
X
at the point P due to the current flowing Y -axis at a distance of 1.0 m I
through XY ? ∆l
due to the element ∆x = w cm.
X M Also, give the direction of Z

I
the field produced.
P
19. If a current loop of radius R carrying an
anti-clockwise current I is placed in a plane
N
Y ISC 2012 parallel to YZ-plane.Then, what will be the
magnetic field at a point on the axis of the
12. Name the kind of magnetic field produced by
loop?
an infinitely long current carrying conductor.
20. A circular coil of closely wound N turns and
13. Draw the magnetic field lines due to a current radius r carries a current I in the clockwise
carrying loop. direction. Find
14. An electron is revolving around a circular loop (i) the direction of magnetic field at its centre.
as shown in the figure. What will be the (ii) the magnitude of magnetic field at the
direction of magnetic field at the point A? centre.
e–
21. A straight wire of length L is bent into a
semi-circular loop. Use Biot-Savart’s law to
A
deduce an expression for the magnetic field at
its centre due to the current I passing
15. There is a circuit given below, where APB and through it.
AQB are semi-circles. What will be the magnetic 22. A wire of length L is bent round in the form of a
field at the centre C of the circular loop? coil having N turns of same radius. If a steady
P current I flows through it in clockwise
direction, then find the magnitude and direction
A of the magnetic field produced at its centre.
C
I B
23. Two identical circular loops P and Q, each of
Q radius r and carrying equal currents are kept in
the parallel planes having a common axis passing
Short Answer Type I Questions through O. The direction of current in P is
clockwise and in Q is anti-clockwise as seen from
2 Marks
O, which is equidistant from the loops P and Q.
16. State Biot-Savart’s law. ISC 2014, 05, 01 Find the magnitude of the net magnetic field at O.
2r
17. State Biot-Savart’s law. A current I flows in a
conductor placed perpendicular to the plane of I
the paper. Indicate the direction of the magnetic O
field due to a small element dl at a point P P Q
situated at a distance r from the element as
shown in the figure. Short Answer Type II Questions
Z 3 Marks
I
24. Use Biot-Savart’s law to derive the expression
dl for the magnetic field on the axis of a current
O Y
r P carrying circular loop of radius R. Draw the
magnetic field lines due to a circular wire
X carrying current I.
170 |

25. Using Biot-Savart’s law, write the expression for


31. An element dl = dx i$ (where, dx = 1 cm) is placed
the magnetic field B due to an element dl
at the origin and carries a large current I = 10 A.
carrying current I at a distance r from it in a
What is the magnetic field on the Y -axis at a
vector form.
distance of 0 . 5 m? NCERT (2 M)
Hence, derive the expression for the magnetic
field due to a current carrying loop of radius R 32. A long straight wire in the horizontal plane
at a point P and distance x from its centre along carries a current of 50 A in North to South
the axis of the loop. direction.Give the magnitude and direction of B
26. Obtain an expression for magnetic flux density B at a point 2.5 m East of the wire. NCERT (2 M)
at the centre of a circular coil of radius R having 33. (i) Two infinitely long X Y
N turns and carrying a current I. ISC 2015 current carrying
27. Obtain an expression for magnetic flux density conductors X and Y are
5A Vacuum 7A
B at the centre of a circular coil of radius R and kept parallel to each
having N turns, when a current I is flowing other, 24 cm apart in
through it. ISC 2009, 01 vacuum. They carry 24 cm
currents of 5A and 7A respectively, in the
Long Answer Type II Questions same direction, as shown in figure below.
Find the position of a neutral point, i.e. a
5 Marks point where resultant magnetic flux
28. State Biot-Savart’s law expressing it in the density is zero. (Ignore earth’s magnetic
vector form. Use it to obtain the expression for field).
the magnetic field at an axial point distance d (ii) If current through the conductor Y is
from the centre of a circular coil of radius a reversed in direction, will neutral point lie
carrying current I. Also, find the ratio of the between X and Y , to the left of X or to the
magnitudes of the magnetic field of this coil at right of Y ? ISC 2016 (2 M)
the centre and at an axial point for which
d = a 3. 34. Two wires A and B have the same length equal
to 44 cm and carry a current of 10 A each. Wire
29. Two very small identical circular loops (1) and A is bent into a circle and wire B is bent into a
(2) carrying equal current I are placed vertically square.
(with respect to the plane of the paper) with
(i) Obtain the magnitudes of the fields at the
their geometrical axes perpendicular to each
centres of the two wires.
other as shown in the figure. Find the
magnitude and direction of the net magnetic (ii) Which wire produces a greater magnetic
field produced at the point O. field at its centre? (3 M)

35. Calculate the magnetic field (in SI unit) at the


(1) O centre of a tangent galvanometer coil of
I
50 turns and radius 10 cm when a current of 1.0
90°
A passes through it. ISC 2007 (2 M)

x 36. A tightly wound 100 turns coil of radius 10 cm is


carrying a current of 1 A . What is the magnitude
of the magnetic field at the centre of the coil?
(2) NCERT (2 M)

I 37. The wire shown in the figure C


carries a current of 10 A.
Numerical Problems Determine the magnitude of
I
magnetic field induction at O I
30. A current of 5 A is flowing from South to North
the centre O. Given the 90°
in a straight wire. Find the magnetic field due
radius of bent coil is 3 cm.
to a 1cm piece of wire at a point 1 m North-East (2 M) A B
from the piece of wire. (2 M)
Moving Charges and Magnetism | 171

38. Two identical circular coils, P and Q each of Y


radius R, carrying currents 1 A and 3 A P
respectively, are placed concentrically and
perpendicular to each other lying in the XY and 0.5 m

YZ-planes. Find the magnitude and direction of


X
the net magnetic field at the centre of the coils. ∆x
(2 M)
10−7 × 10 × 10−2
i.e. |dB| = −2
= 4 × 10−8 T
39. A straight wire carrying a current of 10 A is bent 25 × 10
into a semi-circular arc of radius 2.0 cm as As, dl × r = ∆x$i × y$j = y ∆x ( $i × $j) = y ∆xk$
shown in the figure. What is the magnetic field
So, the direction of the field is in the + Z -direction.
at O due to
µ Idl × r µ Idl sin θ
(i) straight segments? 3. (b) | dB | = 0 = 0 ×
4π r3 4π r2
(ii) the semi-circular arc? (3 M)
If point lies on the conductor, then θ = 0° or 180° and
sin θ = 0, then dB = 0. Hence, the magnetic field induction
at any point on the conductor itself is zero.
µ πi 22 × 12
O 4. (c) B = 0 = 10−7 × = 1.9 × 10−4 T
40. Two concentric circular coils x and y of 4π R 7 × 2 × 10−2
radii 16 cm and 10 cm respectively lie in the Thus, B is 1.9 × 10−4 T normal to the plane of the paper
same ver tical plane containing the North to going into it.
South direction. Coil x has 20 turns and carries 5. (c) Magnetic field at any point lying on the
a current of 16 A , coil y has 25 turns and carries current-carrying straight conductor is zero.
a current of 18 A . The sense of the current in x is Here, H 1 = magnetic field at M due to current in PQ
anti-clockwise and clockwise in y, for an and H 2 = magnetic field at M due to QR + magnetic
observer looking at the coils facing West. Find field at M due to QS + magnetic field
at M due to PQ.
the magnitude and direction of the net
H 3 H 2
magnetic field due to the coils at their centre. = 0 + 1 + H1 = H1 ⇒ 1 =
NCERT (4 M) 2 2 H2 3

41. Two identical loops P and Q Q 6. A current carrying wire produces magnetic field but
each of radius 5 cm are wire which does not carry current has no magnetic field.
lying in perpendicular 7. It can be justified by placing a magnetic needle around
planes such that they have P current carrying wire, which shows deflection of needle.
a common centre as shown 8. Magnetic field produced by a current conductor is
in the figure. Find the (i) Directly proportional to the current flowing through
magnitude and direction of the conductor, length of the element and sine of the
angle between the length of the element and line
the net magnetic field at the common centre of joining the element to the point (1/2)
the two coils, if they carry currents equal to 3 A (ii) Inversely proportional to the square of the distance
and 4 A, respectively. (3 M) between the element and the point. (1/2)
9. Biot-Savart’s law states that, the magnitude of magnetic
field intensity (dB) at a point P due to current element is
Hints & Solutions given by
I dl sinθ
1. (b) Biot-Savart’s law in vector form is dB ∝
r2
µ I ( dl × r )
dB= 0 µ I dl sinθ
4π r3 or dB = 0
4π r2
Hence, option (b) is correct.
Thus, in vector notation,
2. (a) The magnitude of magnetic field, µ Idl×r
µ I dl sin θ dB = 0
| dB | = 0 4π r 3 (1)
4π r2
172 |

10. Biot-Savart’s law is an angle dependent law. (1) 19. Refer to text on page 166 (Magnetic field on the axis of a
11. According to Biot-Savart’s law, we can say that the circular current carrying loop). (2)
direction of magnetic flux density at the point P due to 20. (i) Inward (1)
the current flowing through the conductor XY is (ii) Refer to text on page 167 (Magnetic field at the centre
perpendicular to the plane of the paper directed into the of a current carrying circular loop). (1)
page, as shown in the diagram. 21. According to the questions the wire will now look like.
X
dl

B I B r
P r
r C
Y (1)
Q Length L is bent into semi-circular loop.
12. An infinitely long current carrying conductor produces
magnetic field in the form of concentric circular loops in ∴ Length of wire = Circumference of semi-equal circular wire
a plane of straight conductor. (1) ⇒ L = πr
L
13. Magnetic field lines due to a current carrying loop is ⇒ r= …(i)
π (1/2)
given by
Considering a small element dl on current loop. The
N
magnetic field dB due to small current element Idl at
centre C. Using Biot-Savart’s law, we have
µ Idl sin 90 °
dB = 0 ⋅ [Q Idl ⊥ r , ∴ θ = 90 ° ]
4π r2 (1/2)
µ 0 Idl
dB = ⋅
S (1) 4π r 2
14. As, electron is revolving clockwise, therefore ∴ Net magnetic field at C due to semi-circular loop,
conventional current due to the motion of electron will µ 0 Idl µ 0 I
B=∫ = dl
be in anti-clockwise direction. semi-circle 4 π r 2 4 π r 2 ∫ semi-circle (1/2)
So, according to right hand rule, magnetic field at point µ I  L
A will be in outward direction. = 0⋅ 2 L  but r = 
4π r  π
15. Magnetic field due to loop APB at the centre is given by µ0 IL µ 0 IL
µ I = ⋅ = × × π2
B1 = 0 u 4 π ( L / π )2 4 π L2
4a
µ Iπ
Magnetic field due to loop AQB at the centre is given by = 0
µ I 4L
B2 = 0 u ⋅ which is the required expression. (1/2)
4a
So, net magnetic field at centre = B1 + B 2 = 0 (zero). (1) 22. When a straight wire is bent into the I
form of a circular coil of N turns, then
16. Refer to text on page 164 (Magnetic field due to a
the length of the wire is equal to
current element : Biot-Savart’s law). (2)
circumference of the coil multiplied by
17. Refer to text on page 164 (Magnetic field due to a the number of turns. Let the radius of
current element : Biot-Savart’s law). (2) coil be r.
18. Biot-Savart’s law states that As, the wire is bent round in the form of a coil having N
µ Id l × r$ turns.
dB = 0 ⋅
4 π | r |2 (1) ∴ N × circumference of the coil = Length of the wire
Here ∆x = w cm ⇒ ( 2πr ) × N = L
L
Q ∆l = ∆xi ⇒ r= …(i)
⇒ I = 2A, r = 1 m 2πN
µ ( 2w$i × $j) Magnetic field at the centre due to N turns of a coil is given
∴ dB = 0 ⋅ by
4π (1 )2 µ ( NI ) µ ( NI )
Idl = 2 × w $i B= 0 = 0 [from Eq. (i)]
(1/2) 2r  L 
2 
Q r$ = $j  2 πN 
⇒ | r| = 1 m µ 0 πN 2 I
µ w µ w =
∴ dB = 0 k$ ⇒ | dB | = 0 L (1½)
2π 2π The direction of magnetic field is perpendicular to the plane
and direction along +Z -axis. (1/2) of loop and entering into it. (1/2)
Moving Charges and Magnetism | 173

23. Magnetic field at O due to two loops will be in same Consider a small element dl on circular coil of radius R
direction (Q → P , along the axis) and of equal carrying current I .
magnitude. (1/2)
B = B1 + B2 but B2 = B1
 µ 0 Ir 2  dB
⇒ B = 2 B1 = 2  2 2 3 /2  90° θ = 90°
 2 (r + r )  (1/2) I R
dl
µ 0 Ir 2 µ 0 Ir 2
= =
( 2r 2 )3 /2 23 /2 r 3 (1/2) (1)
µ 0I ∴ By Biot-Savart’s law, magnetic field at the centre due
= 3 /2 to element of coil,
2 r (1/2)
24. Refer to text on page 166 (Magnetic field on the axis of a
circular current carrying loop).
R
For magnetic filed lines, refer to solution 13 on page 172. dB
(3)
Q
25. Refer to text on pages 164 and 166 (Magnetic field due to
a current element : Biot-Savart’s law and magnetic field
on the axis of a circular current carrying loop). (3) µ 0 I dl sin 90°
dB = ⋅
26. Consider a circular coil of radius R and carrying a 4π R2
current I in the direction as shown in figure. Suppose, µ I
the entire circular coil is divided into a large number of ⇒ dB = 0 ⋅ 2 dl
4π R (1)
current elements, each of length dl.
µ0 I µ0 I µ0 I
B=∫ dl = dl = × 2π R
4π R2 ∫
According to Biot-Savart’s law, the magnetic field dB at
4π R2 4π R2
the centre O of the coil due to the current element Idl is
given by µ I
B= 0 [Q ∫ dl = 2πR]
µ I (d l × R ) 2R
B= 0 µ NI
4π R3 (1) For N turns of coil B = 0 .
2R (1)
The magnitude of d B at the centre O is 28. Refer to text on pages 164 and 166 (Magnetic field due to
µ Idl × R sin θ µ Idlsin θ a current element : Biot-Savart’s law and magnetic field
dB = 0 ⋅ 3
⇒ dB = 0
4π R 4π R2 on the axis of a circular current carrying loop). (2)
Magnetic field at the axial point distance from the centre,
µ 2 πa 2 I
I B1 = 0 ⋅ 2
4 π ( a + d 2 )3 /2
90°
In this answer, put r = d .
R
Magnetic field induction at the centre of the circular coil
carrying current is
µ 2 πI B a2 × a
dB B2 = 0 ⋅ ⇒ 1 = 2
4π a B2 ( a + d 2 )3 /2
µ Idlsin θ
∴ B = ∫ dB = ∫ 4 π0 R2 a3 a3
= = [ Qd = a 3]
θ = 90° ⇒ sin90° = 1 (1) ( a 2 + d 2 ) 3 / 2 ( a 2 + 3a 2 ) 3 / 2
µ 1 a3 a3
∴ B = 0 2 ∫ dl = =
4π R ( 4 a 2 )3 /2 8 a3
dl = total length of the coil = 2πr B1 1
∫ ⇒ =
B2 8
µ 1 µ I (3)
B = 0 2 ( 2π R ) = 0 29. The magnetic field at a point due to a circular loop is
4π R 2R
If the coil has N turns. given by
µ NI µ 2πIa 2
∴ B= 0 B= 0⋅ 2
2R (1) 4 π (a + r 2 )3 /2 (1)
27. Take a small current element in the circular coil and where, I = current through the loop,
find B due to it. After that integrate it for whole length a = radius of the loop
of the loop.
and r = distance of O from the centre of the loop.
174 |

Since I , a and r = x are the same for both the loops, the Let at point P, the magnetic flux density be zero.
magnitude of B will be the same and is given by So, magnetic field due to 5 A wire at point P is
2πIa 2 µ 05
µ
B1 = B 2 = 0 ⋅ 2 ( − k$ ) ... (i)
(1) 2π x
4 π (a + x 2 )3 /2
Magnetic field due to 7A wire at point P is
The direction of magnetic field due to loop (1) will be
µ 07
away from O and that of the magnetic field due to loop ( + k$ ) ... (ii)
(2) will be towards O as shown. The direction of the net 2π( 24 − x )
magnetic field will be as shown below We know that, P is a neutral point, so from
Eqs. (i) and (ii), we get
Bnet
B2 5µ 0 7µ 0
=
2πx 2π( 24 − x )
(1) B1 O
I 5 7
⇒ =
90° x 24 − x
⇒ 120 − 5x = 7 x ⇒ 12x = 120
x ⇒ x = 10 cm (from current carrying wire X)
Thus, the position of a neutral point is 10 cm. (1½)
(2) (ii) No, on reversing the current in the wire Y , neutral
point will not lie between X and Y . It will lie on the
I (2) left of wire X. (1½)
The magnitude of the net magnetic field is given by 34. Given, I = 10 A, length of each wire = 44 cm
µ 2 π 2Ia 2
Bnet = B12 + B22 = 0 ⋅ 2 (i) Let r be the radius of wire A when it is bent into a
4 π ( a + x 2 )3 /2 (1) circle.
30. Here, I = 5 A , dl = 1 cm = 001
. m, r = 1 m, θ = 45° 7
⇒ 2πr = 44 ⇒ r = m
[Q direction is North-East] 100
µ 0 Idl sin θ Magnetic field at the centre of the circular coil
∴ dB = ⋅ carrying current is given by
4π r2 µ 2π I 22 100
5 × 001
. × sin 45° B= 0⋅ = 10−7 × 2 × × 10 × = 9 × 10−5 T
= 10−7 × (1) 4π r 7 7
(1 )2 When another wire is bent into a square of each
= 3.54 ×10−9 T side L, then
Its direction is vertically downwards. (1) 4 L = 44 ⇒ L = 11cm = 0.11m
31. Here, dl = dx = 1 cm = 10 −2
m Since, magnetic field induction at a point, at
perpendicular distance a from the linear conductor
I = 10 A, r = 05. m carrying current is given by
Using Biot-Savart’s law, µ I
B = 0 (sin θ1 + sin θ 2 )
µ Id l × r 4 πa
dB = 0 ⋅
4π r3 (1) µ I
B = 4 × 0 (sin 45° + sin 45° )
µ Idx µ Idx 4 πa
= 0 ⋅ 2 ( $i × $j) = 0 ⋅ 2 k$ [Q $i × $j = k]
$
10  1 1 
4π r 4π r = 4 × 10−7 × +
 
10−7 × 10 × 10−2 $ (11 /100)  2 2
= k
(0.5)2 . × 10−5 T
=103 (2½)
(ii) The magnetic field due to a square will be more than
= 4 × 10−8 k$ T (1) that due to a circle of same perimeter. (1/2)
−6
32. Refer to example 1 on page 165. [Ans. 4 × 10 T] 35. Magnetic field at the centre of tangent galvanometer is
33. (i) According to the question, given by
µ NI
X Y B= 0
2r
Vacuum Given, N = 50, r =10 cm , I = 1 A
5A 7A Also, µ 0 = 4 π × 10−7 (1)
x 4 × 3.14 × 10−7 × 50 × 1
P (24 – x) cm So, B=
2 × 0.1
24 cm = 3.14 × 10−4 T (1)
Moving Charges and Magnetism | 175

36. Refer to example 4 on page 167. [Ans. 6.28 × 10−4 T] 1 µ 0I  µ 0I (4 π × 10− 7 ) × 10


=   = =
−2
37. Here, I = 10 A, r = 3 cm, r = 3 × 10 m 2  2r  4r 4 × 2 × 10− 2
Angle subtended by coil at the centre, . cm = 2 × 10−2 m]
[given, I = 10 A and r = 20

θ = 360° − 90° = 270° = rad (1/2) = 5π × 10− 5 T (1)
2
Magnetic field induction at O due to current through 40. For coil x, North
Radius of coil, rx = 16 cm = 0.16m x
circular path ACB is y Ix = 16 A
µ I 10 3π Number of turns, nx = 20 Iy =18 A
B = 0 ⋅ θ = 10 − 7 × ×
4π r ( 3 × 10−2 ) 2 Current in the coil, I x = 16 A West
O
East
(1½)
(anti-clockwise)
= 1.57 × 10− 4 T
For coil y,
Q
38. Magnetic field due to circular wire P, Bnet Radius of coil, ry = 10 cm = 0.1m South
µ 2 π I1 B P
BP = 0 × BQ P Number of turns, ny = 25
4π R
I Current in the coil, I y = 18 A (clockwise)
[along vertically upwards]
µ 0 I1 The magnitude of the magnetic field at the centre of coil x,
= µ 2I π n
2R Bx = 0 ⋅ x x
Magnetic field due to circular wire Q, 4π rx
µ
BQ = 0 ×
2 πI 2
[along horizontal towards left] 10− 7 × 2 × 16 × π × 20
=
4π R 0.16
µ I −4
= 0 2 = 4 π × 10 T (1)
2R (1)
Net magnetic field at the common centre of the two coils, The direction of magnetic field due to the coil x at centre
O is towards right, i.e. East, according to right hand
B = BP2 + BQ2 thumb rule. The magnitude of the magnetic field at the
2 2 2 centre of coil y, (1/2)
µ I  µ I  µ 
=  0 1  +  0 2  =  0  ( I12 + I 22 ) µ 0 2π I y ny
 2R   2R   2R  By = ⋅
4π ry
µ0 4 π × 10 −7
= I12 + I 22 = (1)2 + ( 3 )2 10−7 × 2 × π × 18 × 25
2R 2×R = = 9π × 10−4 T
0.1 (1/2)
4 π × 10 −7
= T The direction of magnetic field due to coil y at centre O is
R
towards left, i.e. West, according to right hand thumb
Resultant magnetic field makes an angle θ with direction of rule. Here, the magnitude of B y is greater than B x , so the
BQ , which is given by resultant magnetic field will be in the direction of B y , i.e.
B 1 left (West). (1)
tan θ = P =
BQ 3 Net magnetic field at the centre,
∴ θ = 30 ° (1) B = By − Bx
39. (i) Magnetic field due to straight segment is = (9π − 4 π ) 10−4 T = 5π × 10−4 T
µ I dl × r [Q B y and B x are opposite to each other]
B=∫ 0⋅
4π r3 = 1.6 × 10−3 T [towards West] (1)
41. Refer to solution 27 on page 173.
r 2 2
Q µ i   µ 0iQ 
P
r
R S Bnet = BP2 + BQ2 =  0 P  +  
O  2r   2r  (1)
For point O, d l and r for each element of straight
µ 4 π × 10 −7
segments PQ and RS are parallel. = 0 iP2 + iQ2 = ×5
Therefore, d l × r = 0. 2r 2 × 5 × 10 −2
Thus, magnetic field due to straight segment is zero. = 2 π × 10 −5 T (1)
(2) Resultant magnetic field makes an angle θ with BQ which is
(ii) Magnetic field at centre O due to semi-circular arc given by,
Magnetic field at centre of circular coil B i 3
= tan φ = P = P =
2 BQ iQ 4 (1)
176 |

TOPIC Ampere’s Circuital Law


02 and Moving Charges

Ampere’s Circuital Law ⇒ B ∫ dl = µ 0 I


According to this law, the line integral of a magnetic field µ 0I
B around any closed path in vacuum is µ 0 times the net ⇒ B × 2 πr = µ 0 I or B=
2 πr
current I net enclosed by the curve.
Example 1 A straight wire carries a current of 3 A.
C
Calculate the magnitude of the magnetic field at a point
I I
15 cm away from the wire.
Sol. Here, current, I = 3 A, point where magnetic field is to
be determined, a = 15 cm = 0.15 m
Surface
Boundary µ 0 2I
∴ Magnitude of magnetic field, B =
4 πa
Mathematically, ∫ B ⋅ d l = µ 0 I net 10−7 × 2 × 3
= = 4 × 10− 6 T
Ampere’s law is applicable only for an Amperian loop 0.15
as the Gauss’s law is used for Gaussian surface in
electrostatics.
The choice of an Amperian loop has to be such that, at
Solenoid and Toroid
each point of the loop either A solenoid is an insulated long wire closely wound in the
(i) B is tangential to the loop and is a non-zero constant form of a helix. Its length is very long as compared to its
(ii) B is normal to the loop diameter. A toroid is a hollow circular ring on which a
(iii) B vanishes large number of insulated turns of a metallic wire are
closely wound. Solenoid and toroid are two equipments
Ampere’s circuital law has same content as the
which are used to produce magnetic fields.
Biot-Savart’s law. Both of these relate magnetic field and
current and express the same physical consequences of a In television, we make use of solenoid to generate
steady electrical current. Ampere’s circuital law holds for magnetic field needed for the deflection of electrons in
any loop but does not always facilitate. Ampere’s circuital picture tube.
law can be conveniently applied in situations of high Toroid is used in devices such as synchroton in which
symmetry. e.g. To find magnetic field of a straight wire, high magnetic field is required, which is generated by
magnetic field of solenoid and toroid as discussed in either a toroid or combination of both solenoid and
coming sections. toroid.
Magnitude of Magnetic Field of a Both solenoid and toroid have symmetrically geometric
shapes, therefore we can apply Ampere’s law conveniently
Infinite Long Straight Wire using to find the magnetic field.
Ampere’s Circuital Law
Magnetic field due to a straight B
Magnetic Field of a Solenoid
conductor at a point P at a distance (r ) A long coil of wire consisting of closely packed loops is
dl
is in the form of a circle of radius (r ) called solenoid, whose magnetic field resembles that of a
which is taken as closed path for r bar magnet of south(S) and north(N) poles as shown in
Amperian loop. Angle between B and O P the figure given below.
d l is zero, everywhere in this path.
Hence, on applying Ampere’s law to
this closed path, we get Amperian
I loop
∫ B⋅dl = µ 0I I Solenoid II

Magnetic field of a solenoid


or ∫ B dl cos0° = µ 0 I
Moving Charges and Magnetism | 177

Inside the solenoid, magnetic field is uniform and parallel to Note The formula for magnetic field B = µ 0 nl is only valid when
the solenoid axis. Outside the solenoid, magnetic field is length of the solenoid (l) is much larger than its radius(r), i.e.
l >> r.
assumed to be zero.
Consider an air cored solenoid having closely packed coils Example 2 The length of a solenoid is 0.2 m and it
in which I is current, n is number of turns per unit has 120 turns. Find the magnetic field in its interior, if a
length and B is magnetic field inside the solenoid. current of 2.5 A is flowing through it.
Applying Ampere’s circuital law to determine magnetic Sol. Here, l = 0.2 m, N = 120, I = 25
. A
field (B) inside the solenoid, we choose rectangular closed Magnetic field in the interior of the solenoid,
path PQRS, where PQ = L and the line integral of B over N
B = µ 0nI = µ 0 I
closed path PQRS is l
Q R −7 120
= 4 π × 10 × × 2 .5
∫ PQRS
B⋅ d l = ∫P B ⋅ d l+ ∫Q B ⋅ d l 0.2
S P
= 1.85 × 10−3 T
+ ∫ R B ⋅d l + ∫ S B ⋅d l
Example 3 A solenoid of length 0.5 m has a radius
R P
Now, ∫Q B ⋅d l = ∫ S B ⋅d l = 0 of 1 cm and is made up of 500 turns. It carries a current
of 5 A. What is the magnitude of magnetic field inside
[because along QR and PS, the field B is at right angles to the solenoid? NCERT
dl, so that B .d l = Bdl cos 90° = 0] Sol. Given, total number of turns, N = 500
S
Length of solenoid, l = 0.5 m
∫ R B ⋅d l = 0 [Q B is zero at points outside the solenoid] Current, I = 5 A
Q Q Q
∴ ∫ B ⋅ d l = ∫ B ⋅ d l = ∫ B dl cos0° = ∫ B dl = BL and radius r = 1 cm = 10 −2 m
PQRS P P P l 0.5
Here, = = 50 ⇒ l >>r
S R r 10−2
µ NI 500
∴ B = µ 0nI = 0 = 4π × 10− 7 × ×5
l 0.5
P Q = 6.28 × 10− 3 T
L

Magnetic Field of a Toroid


Hence, from Ampere’s law, An endless solenoid in the form of a ring is called a toroid.
∫PQRS B ⋅ d l = µ 0 × (current enclosed by PQRS) Magnetic field lines inside the toroid are circular,
concentric with the centre of toroid.
Here, number of turns per unit length is n, the number Let I be the current, r be the mean radius, n be the number
of turns in length L is nL. The current in each turn is I, of turns per unit length and B be the magnetic field inside
so net current enclosed by the loop is nLI. the toroid.
Total current, I net = nLI Q
r3
B
∴ ∫ B ⋅ d l = µ 0 × nIL
PQRS Q S
3
⇒ BL = µ 0nIL 2
P
⇒ B = µ 0nI
I
From the expression, it is clear that B is independent of r2
the length and diameter of the solenoid and is uniform
over the cross-section of the solenoid.
If a material of permeability µ r is used as a core, then B (a) (b)
inside the solenoid is µ 0µ r n I . (a) A toroid carrying a current I (b) A sectional view of the
toroid. The magnetic field can be obtained at an arbitrary
At points near the end of air closed solenoid, distance r from the centre O of the toroid by Ampere’s
1 circuital law. The dashed lines labelled 1, 2 and 3 are three
B= µ 0nI circular Amperian loops
2
178 |

The line integral of magnetic field around closed path of because in this case the angle θ = 90°, in the expression
circle of radius r is F = qvB sin θ and the maximum force will be
F max = q vB sin 90° = qvB [Qsin 90° =1]
∫ B ⋅ d l = ∫ B d l cos 0° = B × 2πr
The magnetic force includes the cross product of velocity
Now, from Ampere’s law, ( v ) of the particle and magnetic field (B). Thus, the
∫ B ⋅ d l = µ 0 × current enclosed by closed path magnetic force will be zero, if the velocity vector and
magnetic field vector are either parallel or anti-parallel to
⇒ B × 2 πr = µ 0 n (2 πr ) I or B = µ 0 nI each other.
If the toroid is a material cored of relative permeability The force (F magnetic ) acting on a charged particle moving
µ r , then magnetic field inside the toroid, with velocity ( v ) through a magnetic field ( B) is always
B = µ 0µ r n I perpendicular to v and B.
From right hand thumb rule, the force F is perpendicular
Example 4 A toroid has a core of inner radius 25 cm to velocity (v) and magnetic field (B). Hence, it changes
and outer radius 26 cm, around which 3500 turns of a its path continuously.
wire are wound. If the current in the wire is 11 A . What
is the magnetic field (i) outside the toroid (ii) inside the
Motion of a Charge in Magnetic Field
core of toroid (iii) in the empty space surrounded by the In case of motion of a charge in a magnetic field, the
toroid? NCERT magnetic force is perpendicular to the velocity of the
Sol. Here, I = 11 A, total number of turns = 3500
particle. So, no work is done and no change in the
magnitude of the velocity is produced.
25 + 26
Mean radius of toroid, r = = 25.5 cm B
2
= 25.5 × 10−2 m q
+ v
Total length of the toroid = 2 πr = 2 π × 25.5 × 10−2 v F r
F
= 51 π × 10 − 2 m +
q
Therefore, number of turns per unit length F
3500 p
n= +
qv
51π × 10 −2
Circular motion
(i) The field is non-zero only inside the core surrounded
by the windings of the toroid. Therefore, the field Magnetic force acts as a centripetal force and produces a
outside the toroid is zero. circular motion perpendicular to the magnetic field. If v
(ii) The field inside the core of the toroid, and B are perpendicular to each other, then particle will
3500 describe a circle.
B = µ 0nI = 4 π × 10−7 × × 11
51π × 10−2 As, centripetal force required for circular motion is
= 3.02 × 10−2 T provided by magnetic force, so
(iii) For the reason given in (i), the field in the empty space mv ⊥2 mv p
surrounded by toroid is also zero. = qBv ⊥ or r = ⊥ = ...(i)
r qB qB
Force on a Moving Charge in a Uniform where, m = mass of charged particle
Magnetic Field and r = radius of circular path.
When a charged particle (q) moves with velocity (v) where, v ⊥ and v || are perpendicular and parallel
inside a uniform magnetic field B, then force acting on components of the velocity v.
it is F = q ( v × B)
Radius of the path of the charged particle is proportional
Force due to magnetic field depends on q, v , B. The to the momentum ( p = mv ) of the particle and inversely
magnetic force will be zero, if the particle is at rest. The proportional to the magnitude of charge (q) and magnetic
reason is that for the charged particle at rest | v | = 0, field ( B ).
which will turn the expression for magnetic force, In terms of kinetic energy ( K ), the equation may be
q ( v × B), into zero. Only moving charges feel the
magnetic force.The magnetic force is at its maximum 2mK
expressed as, r= [Q p = 2mK ]
value, when v and B are perpendicular to each other qB
Moving Charges and Magnetism | 179

Time period (T ) of the motion of a charge is given by mv 2mqV 2mV


Radius, r = or r = =
2πr 2π mv qB qB qB 2
T = = ×
v v qB rp 2mV 2eB 2 1
Thus, = × =
2πm rα eB 2 2( 4m ) V 2
or T = ...(ii)
qB
Example 6 A beam of protons with a velocity of
2 π Bq
Angular frequency, ω = = 4 × 10 5 ms −1 enters in a region of uniform magnetic
T m field of 0.3 T. The velocity makes an angle of 60° with
Bq
Frequency, ν= [Qω = 2 πν ] the magnetic field. Find the radius of the helical path
2 πm taken by the proton beam and the pitch of the helix.
The angular speed ω of the particle is given by Sol. Velocity component along the field
ω = v /r or ω = (qB )/m v || = 4 × 105 × cos 60° = 2 × 105 ms −1
and velocity component perpendicular to the field.
Helical Motion
v ⊥ = ( 4 × 10 5 ) sin 60 ° = 2 3 × 10 5 ms −1
If a charged particle has a velocity not perpendicular to
Proton will describe a circle in plane perpendicular to
B, then component of velocity along B remains magnetic field with radius,
unchanged as the motion along the magnetic field will mv ⊥ (1.67 × 10 −27 kg ) × ( 2 3 × 10 5 ms−1 )
not be affected by the magnetic field. Then, the motion r= = = 1. 2 cm
qB . × 10 −19 C ) × (0. 3 T )
(16
of the particle in a plane perpendicular to B is as before a
circular one, thereby producing a helical motion. Time taken to complete one revolution is
2 πr 2 × 314 . × 0.012
Y T = = s
v⊥
v⊥ 2 3 × 10 5
v Because of v|| protons will also move in the direction of
⊕ vII magnetic field.
q
∴ Pitch of helix = v|| × T
Z
B 2 × 10 5 × 2 × 314
. × 0.012
Pitch X = m
2 3 × 10 5
Radius
Helical motion
= 0.044 m = 4.4 cm

For helical path, the distance moved along the magnetic Force on a Moving Charge in a Uniform
field in one rotation is called pitch (P).
Magnetic and Electric Field
2πmv ||
P = v || T = (Lorentz Force)
qB Suppose a point charge (q) is moving in the presence of
Note One tesla (1 T) is defined as the field which produces a both electric and magnetic fields. Let q be the magnitude
force of one newton (1 N) when a charge of one coulomb (1 C) of the charge, v be velocity of the point charge, B be the
moves perpendicularly in the region of the magnetic field at a magnetic field and E be the electric field. We have
velocity of
1 ms −1
studied two kinds of forces that can be exerted on an
electrically charged particle.The electric force is given by
Example 5 A proton and an α-particle, accelerated F = q E and the magnetic force is F = q ( v × B).
through same potential difference, enter in a region of The sum of these forces represents the net force that can
uniform magnetic field with their velocities be exerted on a particle due to its electric charge (q), this
perpendicular to the field. Compare the radii of circular sum is called the Lorentz force and is given by
paths followed by them. FLorentz = Felectric + Fmagnetic
Sol. Let mass of proton = m, charge of proton = e = q E + q ( v × B)
Now, mass of α-particle = 4 m, charge of α-particle = 2e
= q [E + ( v × B)]
When a charge q is accelerated by V volts, it acquires a kinetic
energy EK = qV Force on negative charge is opposite to that of positive
∴ Momentum is given by mv = 2mEK = 2mqV charge.
180 |

Cyclotron The key to the operation of the cyclotron is that the


frequency ( f ) at which the proton circulates in the
It is used to accelerate charged particles or ions to high magnetic field (and that does not depend on its speed)
energies. Electric and magnetic fields are used in must be equal to the fixed frequency f osc of the electrical
combination to increase the energy of charged particles. oscillator, or
The working of a cyclotron is based on the fact that f = f osc (resonance condition) …(i)
frequency of revolution of charged particle is not
This resonance condition says that, if the energy of the
dependent on its energy.
circulating proton is to increase, energy must be fed it at
Particle moves inside the two semi-circular discs like a frequency f osc that is equal to the natural frequency f
metal containers D1 and D 2 called dees. at which the proton circulates in the magnetic field.
When the particle moves from one dee to another, it is Combining f = q B /2πm and Eq. (i) allows us to write
acted upon by electric field. the resonance condition as, qB = 2πm f osc …(ii)
Deflection plate The frequency of oscillation of the charged particle from
Magnetic
field out of Exit port the above expression is
the paper Charged qB
particle f osc =
P 2πm
It is also known as cyclotron frequency.
D1 D2
The time period of oscillation of the charged particle is
1 2πm
Oscillator T = ⇒T =
f osc qB
Cyclotron
Time for one revolution of an ion is independent of its
Electric field changes sign alternatively. Therefore, the speed or radius of its orbit.
particle is accelerated by the electric field, which increases
the energy of the particle. For the proton, q and m are fixed. The oscillator
(we assume) is designed to work at a single fixed
The increase in energy increases the radius of the circular frequency f osc . We then tune the cyclotron by varying B
path. Hence, the path is a spiral one. until Eq. (ii) is satisfied and then many protons circulate
Working through the magnetic field, to emerge as a beam.
Suppose a proton injected by source at the centre P of The expression for kinetic energy of the charged particles
the cyclotron in figure, initially moves towards a can be calculated from the fact that the centripetal force
negatively charged dee. It will accelerate towards this dee of the charged particle is provided by the magnetic force,
and enter in it. Once inside, it is shielded from electric f centripetal = f magnetic
fields by the copper walls of dee, i.e. the electric field mv 2 qBR
does not enter the dee. The magnetic field, however, is ⇒ = qvB or v =
not screened by the (non-magnetic) copper dee, so the R m
proton moves in a circular path whose radius, which Thus, the kinetic energy of the charged particle is
depends on its speed, is given by 1 1  qBR 
2
q 2B 2R 2
r = mv /qB KE = mv 2 = m ×   =
2 2  m  2m
Let us assume that at the instant the proton enters into
the centre gap from the first dee, the potential difference Thus, the positively charged particle will gain maximum
between the dees is reversed. Thus, the proton again faces energy when it is at the priphery of the does where R is
a negatively charged dee and is again accelerated. This maximum.
process continues and the circulating proton always being Example 7
in step with the oscillations of the dee potential, until the A cyclotron oscillator frequency is
proton has spiraled out to the edge of the dee system. 10 MHz. What should be the operating magnetic field
Then, a deflector plate sends it out through a portal (i.e. for accelerating α -particle? If the radius of the dees is
an exit port). 50 cm, what is the kinetic energy in MeV of the
α-particle beam produced by the accelerator?
Moving Charges and Magnetism | 181

Sol. Given, ν = 10 MHz = 107 Hz


r0 = 50 cm = 0.50 m
Topic Practice 2
mα = 4.0028 × 1.66 × 10−27 kg
= 6 .645 × 10 −27 kg Multiple Choice Questions
q = 2e = 2 × 1.6 × 10−19 = 3.2 × 10−19 C 1 Mark
Bq 1. A moving electron centres a uniform and
Q ν=
2 πmα perpendicular magnetic field. Inside the magnetic
2πmα ν field, the electron travels along ISC 2015
∴ B=
q (a) a straight line (b) a parabola
2 × 22 6.645 × 10 × 10−27 7 (c) a circle (d) a hyperbola
= × = 1.305 T
7 3.2 × 10−19 2. In the given figure, E and B are magnitudes of
Thus, maximum kinetic energy is given by electric and magnetic fields which are acting on
B 2 q 2r 2 a moving electron as shown. The direction of
Emax =
2mα velocity of electron is along X-axis.
Y
(1.305)2 × (3.2)2 × 10−38 × 0.25
= MeV E
2 × 6.645 × 10−27 × 1. 6 × 10−13
v
= 20.5 MeV – X
B

Uses of Cyclotron Z
It is used to accelerate the charged particles such as Then, force on electron is given by
protons/negatively charged ions. (a) F = q ( E + vB ) $j (b) F = q( E − vB ) $j
The cyclotron is used to bombard nuclei with energetic (c) F = q ( E + vB ) k$ (d) F = q ( E − vB ) k$
particles, ions into solids, and in hospitals to produce
radioactive substances. 3. An electron moves straight inside a charged
parallel plate capacitor. Plates of capacitor have
Limitations of Cyclotron charge density σ. The space between plates is
filled with constant magnetic field of induction B.
(i) Uncharged particles like neutron, can not σ
accelerated by cyclotron.
(ii) Cyclotron can not accelerate electrons because they
σ
have very small mass. When electrons start moving l
at a very high speed, then oscillating electric field Time taken by electron to pass through the
makes them to go quickly out of step. region is
(iii) According to the Einstein’s special theory of eσ ε 0 lB σe ε 0B
(a) (b) (c) (d)
relativity, the mass of a particle increases with ε 0 lB σ ε 0B eσ
increase of its velocity as
4. A cyclotron is used to accelerate charged
m0 particles or ions to high energies. It uses
m=
v2 (a) only electric field
1− 2 (b) only magnetic field
c
(c) Both electric and magnetic fields
[when v → velocity of charge particle] (d) None of the above
Due to change of mass of charge particle, frequency 5. An alternating electric field of frequency ν, is
 Bq  applied across the dees (radius = R) of a
f =  changes, hence frequency of charge
 2πm  cyclotron that is being used to accelerate
particle does not match with the frequency of high protons (mass = m). The operating magnetic
frequency oscillation. Therefore, charge particle can field ( B) used in the cyclotron and the kinetic
not be accelerated with very high velocity energy ( K ) of the proton beam produced by it,
comparable to velocity of light (c). are given by
182 |


(a) B = and K = 2 mπ 2 ν 2 R2 16. Show that the radius of a charged particle
e
entering perpendicularly in a magnetic field is
2πmν
(b) B = and K = m 2 πνR2 directly proportional to its momentum. ISC 2004
e
2πmν 17. A charge q moving along the X-axis with a
(c) B = and K = 2 mπ 2 ν 2 R2
e
velocity v is subjected to a uniform magnetic

(d) B = and K = m 2 πνR2 field B acting along the Z-axis as it crosses the
e
origin O.
6. Cyclotron frequency of an electron circulating (i) Trace trajectory.
in a magnetic field of 1T is (ii) Does the charged particle gain kinetic
(a) 28 MHz (b) 280 MHz energy as it enters the magnetic field?
(c) 2.8 GHz (d) 28 GHz
Justify your answer.
B
Very Short Answer Type Questions
1 Mark
O Y
7. State Ampere’s circuital law. ISC 2005
q
8. Magnetic field lines can be entirely confined X
within the core of toroid, but not within a
straight solenoid. Why? 18. Write an expression in the vector form for the
Lorentz magnetic force F due to a charge
9. A charged particle enters an environment of a moving with velocity v in a magnetic field B .
strong and non-uniform magnetic field varying What is the direction of the magnetic force?
from point to point both in magnitude and
direction, and comes out of it following a 19. Write an expression for Lorentz magnetic force
complicated trajectory. Would its final speed on a particle of charge q moving with velocity v
equal to the initial speed, if it suffered no in a magnetic field B . Show that no work is
collisions with the environment? NCERT
done by this force on the charged particle.

10. When does a moving charged particle not 20. Find the condition under which the charged
experience any force while moving through a particles moving with different speeds in the
uniform magnetic field? ISC 2014 presence of electric and magnetic field vectors
11. Write an expression for the Lorentz force F in can be used to select charged particles of a
vector form. ISC 2016 particular speed.
21. A point charge is moving with a constant
Short Answer Type I Questions velocity perpendicular to a uniform magnetic
2Marks field as shown in the figure. What should be
12. A long solenoid of length L having N turns magnitude and direction of the electric field so
carries a current I. Deduce the expression for that the particle moves undeviated along the
the magnetic field in the interior of the same path?
solenoid. Y
B
13. Obtain with the help of a necessary diagram,
the expression for the magnetic field in the
+q
interior of a toroid carrying current. v

14. Define one tesla using the expression for the


magnetic force acting on a particle of charge q
X
moving with velocity v in a magnetic field B.
15. A particle of charge q and mass m is moving with 22. State the underlying principle of a
velocity v. It is subjected to a uniform magnetic cyclotron.Write briefly, how this machine is
field B directed perpendicular to its velocity. used to accelerate charged particles to high
Show that it describes a circular path. Write the energies?
expression for its radius.
Moving Charges and Magnetism | 183

Short Answer Type II Questions Under what conditions will it move in


(a) a circular path (b) a helical path?
3 Marks
(ii) Show that the kinetic energy of the particle
23. Using Ampere’s circuital law or Biot-Savart’s moving in magnetic field remains constant.
law, show that magnetic flux density B at a
28. (i) Write an expression for the magnetic
point P at a perpendicular distance a from a
force F acting on a charged particle q
long current carrying conductor is given by moving with velocity v in the presence of
 µ  2I magnetic field B .
B =  0
 4π  a (ii) A neutron, an electron and an alpha
[Statement of the laws not required] ISC 2011, 05 particle moving with equal velocities, enter
a uniform magnetic field going into the
24. Explain, how Biot-Savart’s law enables one to plane of the paper as shown in the figure.
express the Ampere’s circuital law in the Trace their paths in the field and justify
integral form, viz. your answer.

∫ B ⋅ d l = µ 0I α
where, I is the total current passing through the
n
surface.
e
25. A long straight solid metal wire of radius R
carries a current I uniformly distributed over its
circular cross-section. Find the magnetic field 29. A uniform magnetic field B is set up along the
at a distance r from the axis of wire (i) inside positive X-axis. A particle of charge q and mass
(ii) outside the wire. m moving with a velocity v enters the field at the
origin in XY -plane such that it has velocity
26. (i) State Ampere’s circuital law expressing it in components both along and perpendicular to the
the integral form. magnetic field B. Trace, giving reason, the
(ii) Two long coaxial insulated solenoids, S 1 trajectory followed by the particle. Find out the
and S 2 of equal lengths are wound one expression for the distance moved by the particle
over the other as shown in the figure. A along the magnetic field in one rotation.
steady current I flows through the inner
30. (i) Obtain the expression for the cyclotron
solenoid S 1 to the other end B, which is
frequency.
connected to the outer solenoid S 2 through
(ii) A deuteron and a proton are accelerated by
which the same current I flows in the
the cyclotron. Can both be accelerated with
opposite direction, so as to come out at the same oscillator frequency? Give reason
end A. If n 1 and n 2 are the number of turns to justify your answer.
per unit length, find the magnitude and
direction of the net magnetic field at a 31. What is the use of a cyclotron? ISC 2009
point (a) inside on the axis and (b) outside
the combined system. Long Answer Type Questions
5 Marks
l r2
B 32. (i) Using Ampere’s circuital law, derive the
A
expression for the magnetic field in the
r1 vector from at a point on the axis of a
I S1 solenoid.
n1 turns S2 (ii) What does a toroid consist of ? Find out
n2 turns the expression for the magnetic field inside
a toroid for N turns of the coil having the
27. (i) Write an expression for the force F acting
average radius r and carrying a current I.
on a particle of mass m and charge q
Show that the magnetic field in the open
moving with velocity v in a magnetic
space interior and exterior to the toroid is
field B. zero.
184 |

33. Answer the following questions. Numerical Problems


(i) A magnetic field that varies in magnitude
36. (i) Using Ampere’s circuital law, obtain an
from point to point but has a constant expression for the magnetic flux density B
direction (East to West) is set up in a at a point X at a perpendicular distance r
chamber. A charged particle enters the from a long current carrying conductor.
chamber and travels undeflected along a (Statement of the law is not required).
straight path with constant speed. What (ii) PQ is a long straight conductor carrying a
can you say about the initial velocity of the current of 3A as shown in figure below. An
particle? electron moves with a velocity of
(ii) A charged particle enters an environment 2 × 107 ms − 1 parallel to it. Find the force
of a strong and non-uniform magnetic field acting on the electron.
varying from point to point both in
magnitude and direction, and comes out of
it following a complicated trajectory.
Would its final speed equal to the initial
speed, if it suffered no collisions with the
environment?
ISC 2017 (3 M)
(iii) An electron travelling West to East enters a
chamber having a uniform electrostatic 37. A solenoid of length 50 cm having 100 turns
field in North to South direction. Specify carries a current of 2.5 A. Find the magnetic
the direction in which a uniform magnetic field (i) in the interior of the solenoid, (ii) at one
field should be set up to prevent the end of the solenoid. (2 M)
electron from deflecting its straight line 38. A solenoid of length 1.0 m and 3.0 cm diameter
path. NCERT has 5 layers of windings of 850 turns each and
34. (i) Deduce the expression for the frequency of carries a current of 5 A . What is the magnetic
field at the centre of solenoid? Also, calculate
revolution of a charged particle in a
the magnetic flux from a cross-section of the
magnetic field and show that it is
magnetic flux solenoid at the centre of solenoid.
independent of velocity or energy of the
(2 M)
particle.
(ii) Draw a schematic sketch of a cyclotron. 39. A magnetic field of 100 G (1 G = 10 −4 T ) is
Explain the essential details of its required which is uniform in a region of linear
construction how it is used to accelerate dimension about 10 cm and area of
the charged particles? cross-section about 10 −3 m 2. The maximum
current carrying capacity of a given coil of wire
35. (i) Draw a schematic sketch of a cyclotron.
is 15 A and the number of turns per unit length
Explain clearly the role of crossed electric
that can be wound round a core is at most 1000
and magnetic fields in accelerating the
turns m −1. Suggest some appropriate design
charge. Hence, derive the expression for
particulars of a solenoid for the required
the kinetic energy acquired by the
purpose. Assume the core is not ferromagnetic.
particles. NCERT (3 M)
(ii) An α-particle and a proton are released
from the centre of the cyclotron and made
40. A toroid has a core (non-ferromagnetic) of
inner radius 24 cm and outer radius 25 cm,
to accelerate.
around which 3500 turns of a wire are wound.
(a) Can both be accelerated at the same
If the current in the wire is 10A , what is the
cyclotron frequency? Give reason to
magnetic field
justify your answer.
(i) outside the toroid?
(b) When they are accelerated in turn,
which of the two will have higher (ii) inside the core of toroid?
velocity at the exit slit of the dees? (iii) in the empty space surrounded by the toroid?
(3 M)
Moving Charges and Magnetism | 185

41. An electron of energy 2000 eV describes a 2. (b) Y


circular path in magnetic field of flux density E
0.2 T. What is the radius of path? FE

Take, e = 1.6 ×10 −19 C, m = 9 × 10 −31 kg. (2 M) v


X
Direction of B
42. An electron beam passes through a region in current FB
which a magnetic field of 2 × 10 −3 T and an Z
electric field 3.4 × 10 4 V/m both acting When electric and magnetic fields are perpendicular to
simultaneously at right angles to each other. If each other.
the path of the electron remains undeflected, So, FE = qE = qE $j, FB = qv × B
calculate the speed of the electrons. If the = q (v$i × Bk$ ) = − qv$j (Q $i × k$ = − $j)
electric field is removed, what will be the radius Therefore, force on electron is given by
of the circular path of electrons? ISC 2002 (4 M)
F = q ( E − vB )$j
43. An electron moving with a velocity of E σ
5 × 107 ms−1 , enters in a magnetic field of 3. (b) Velocity of a particle, v = =
B ε0 B
1 Wb m −2 at an angle of 30°. Calculate the force
∴ Time taken by electron to pass through the region,
on the electron. ISC 2006, 01 (2 M)
l lε 0 B
44. An electron is moving vertically upwards in the t = =
v σ
uniform magnetic field of the Earth. The
velocity of the electron is 2.0 × 106 m/s and the 4. (c) The cyclotron is a machine to accelerate charged
particles or ions to high energies. The cyclotron used
horizontal component of the Earth’s magnetic both electric and magnetic fields in combination to
field is 0.32 × 10 −4 T. Obtain the magnitude and increase the energy of charged particles.
direction of the force on the electron. eB
5. (c) Frequency, ν =
2π m
1 2 mv
KE = mv and radius, R =
2 eB
πR
Here, velocity, v =
T /2
2π R
= 2π Rν
=
ISC 2000 (2 M) T
m( 2πRν )
45. A cyclotron when being used to accelerate ∴ Radius, R =
eB
positive ions? (Mass = 6.7 × 10 −27 kg, 2πmν
Magnetic field, B =
charge = 3.2 × 10 −19 C) has a magnetic field of e
( π/2) T. What must be the value of the frequency 1
Kinetic energy, K = m(v )2
of the applied alternating electric field to be 2
used in it? (2 M) = 2mπ 2 ν 2 R 2
6. (d) Frequency of an electron,
Hints & Solutions f =
Bq
2π m
1. (c) According to right hand thumb rule, the electron 1 × 1.6 × 10−19
travels along a circle. Since, the right hand thumb rule =
states that, when a force is perpendicular to velocity and 2 × 3.14 × 9.1 × 10−31
hence it changes. Its path continuously and this force = 28 GHz
acts as a centripetal force and produces a circular 7. Refer to text on page 176 (Ampere’s circuital law).
motion perpendicular to the magnetic field.
186 |

8. The magnetic field lines always form closed loops. As, p


⇒ r= ⇒ r ∝p
the turns of the wires in a toroidal solenoid are wound qB
over its core in circular form, the field lines are confined
where, p = mv = momentum.
within the core of toroid. In a straight solenoid, the
magnetic field lines cannot form closed loops within the 17. (i) As, the charged particle is moving perpendicularly to
solenoid. the magnetic field. So, it will perform circular motion
in XY-plane. (1)
9. Yes, the final speed is equal to its initial speed as the (ii) No, the charged particle does not gain any KE as
magnetic force acting on the charged particle only Lorentz force acting on it does not perform any work
changes the direction of velocity of charged particle but as Fm ⊥ v. (1)
cannot change the magnitude of velocity of charged B
particle.
10. Force experienced by a moving charged particle is
| F | = qvB sin θ O Y
where, θ = angle between v and B. q
v X
θ
18. The expression in vector form is given by
B
F = q ( v × B ).
| F | = 0 ⇒ qvBsinθ = 0 The direction of the magnetic force is in the direction of
∴ θ = 0 or π ( v × B ), i.e. perpendicular to the plane containing v
So, for the angle θ = 0 or π force experienced by moving and B (2)
charge is zero. 19. Refer to text on page 179 (Force on a moving charge in a
11. In vector form, expression for Lorentz force can be uniform magnetic and electric field (Lorentz force)). (1)
given as Lorentz magnetic force always acts perpendicular to the
F = Fe + Fm = q E + q ( v × B ) direction of motion of the particle. Thus, work done by
F = q ( E + v × B) this force is zero. (1)

12. Refer to text on pages 176 and177 (Magnetic field of a 20. A diagram in which particle moves in magnetic and
solenoid). electric field is shown below (1)

13. Refer to text on pages 177 and 178 (Magnetic field of a + + + +


B
toroid).
E v> E
F 1N B E
14. Q F = qvB ⇒ B = ⇒ 1T = v=
qv (1C)(1ms−1 ) v B
E
Thus, if a charge of 1 coulomb moving with a velocity of v<
B
– – – –
1 metre per second perpendicular to a uniform magnetic
field experiences a force of 1 newton, then the Forces on a charged particle are
magnitude of the field is 1 tesla. (2)
Fe = electric force = qE ,
15. Refer to text on pages 178 and 179 (Motion of a charge Fm = magnetic force = Bqv
in magnetic field). (1)
For a particle to go straight without any deflection
16. When a charged particle enters in a uniform magnetic
E
field perpendicularly, central force acts on the particle Fe = Fm ⇒ qE = Bqv ⇒ v =
which provides centripetal force for the circular motion B
of the particle. E
In this way, particles having speed, v = are separated.
Situation is shown in figure. B (1)
v 21. As, v = − v $i [Q the particle is moving along X-direction]
F
o
r
B B = − B k$
[Q the magnetic field is perpendicular to the plane of the
2 paper directed inwards, i.e. negative Z-direction]
mv
Centripetal force, F = ∴ Force acting due to magnetic field,
r
Fm = q ( v × B ) = q [ − v$i × ( − B k$ )],
As magnetic force will provide the necessary centripetal
force to the charged particle. F = − qvB$j
m [ Q $i × k$ = − $j ]
(1)
mv 2 mv ⇒ Magnitude of Fm = | Fm | = qvB
⇒ = qvB ⇒ = qB ⇒ p = mv = rqB
r r
Moving Charges and Magnetism | 187

The direction of Fm is along negative Y-direction.For the Let the closed path be made of large number of small
undeflected motion of particle, elements, where
Force due to electric field = Force due to magnetic field, AB = d l1 , BC = d l 2 , CD = d l3
qE = q ( v × B ) Let d θ1 , d θ 2 , d θ 3 , be the angles subtended by the various
∴ E=v×B elements at point O through which conductor is passing.
Magnitude of electric field, | E | = | v × B| and direction of Then, d θ1 + d θ 2 + d θ 3 + K = 2π
magnetic field will be perpendicular to both v and B, i.e. Suppose these small elements AB, BC, CD,…… are small
along Y-axis. (1) circular arcs of radii r 1 , r2 , r3 …… respectively.
22. Refer to text on pages 180 (Cyclotron). d l1 dl2 dl3
Then, dθ1 = , dθ 2 = , dθ 3 =
23. In such cases, where symmetry exist it is easier to find r1 r2 r3 (1)
field by Ampere’s circuital law. If B1 , B 2 , B 3 are the magnetic field inductions at a point
To find magnetic field at a point, consider Amperian along the small elements d l1 , d l 2 , d l 3 ……, then from
loop around an infinitely long conducting wire as Biot-Savart’s law we know that for the conductor of
shown. (1) infinite length, magnetic field is given by
In order to determine the magnetic field at point
µ 0 2I µ 2I µ 2I
P, which is situated at a distance a from the centre of the B1 = ⋅ , B2 = 0 ⋅ , B3 = 0 ⋅
circular loop around the conducting wire. The magnetic 4 π r1 4 π r2 4 π r3
field ( B ) is tangential to the circumference of the loop. In case of each element, the magnetic field induction B and
current element vector d l are in the same direction. (1)
I
Line integral of B around closed path is

a
P B ∫ B ⋅ d l = B1 ⋅ d l 1 + B 2 ⋅ d l 2 + B3 ⋅ d l 3 + K
= B1 ( dl1 ) + B 2 ( dl 2 ) + B 3 ( dl 3 ) + K
Amperian loop µ 2I µ 2I µ 2I
= 0 ⋅ dl1 + 0 ⋅ dl 2 + 0 ⋅ dl 3 +...
4 π r1 4 π r2 4 π r3
As we know that from Ampere’s circuital law,
µ 0 2I  dl1 dl 2 dl 3 
= + + + K
∫ B ⋅dl = µ 0 I 
4 π  r1 r2 r3 
As ∫ dl = 2πa µ 0 2I
= [d θ1 + d θ 2 + d θ 3 + K]
So, B 2π a = µ 0 I 4π
µ I 2 µ 2I µ
B = 0 × ⇒ B= 0 = 0 2I × 2π = µ 0 I
2π a 2 4π a 4π
The direction of magnetic field will be determined by which is an expression of Ampere’s circuital law. (1)
right hand thumb rule. (2) 25. (i) Let the point P be lying inside Y
the wire at a perpendicular I
24. Y distance r from the axis of the B
wire. Consider a circular path
B of radius r around the axis of O r P
dl
r P the wire. By symmetry, the
magnetic field produced due
to current flowing in the wire R
at any point over this path is I
X tangential to it and equal in X
Y
magnitude at all points on this path.
D Current enclosed by the closed path,
C
dθ 3 I 2 Ir 2
dθ 2 B I ′= × π r =
O
dθ1
A πR 2 R2
Applying Ampere’s circuital law,
I ∫ B ⋅d l = µ 0 µr I ′
X
Ir 2
Consider any arbitrary closed path perpendicular to the ⇒ B( 2πr ) = µ 0 µ r
R2
plane of paper around a long straight conductor XY
carrying current from X to Y , lying in the plane of µ µ Ir 2 µ µ Ir
⇒ B = 0 r2 ⇒ B = 0 r2
paper. 2π R r 2π R (2)
188 |

(ii) When point P is outside the wire, r > R, so that the 29. Y
current enclosed by closed path = I B
Using Ampere’s circuital law, ∫ B ⋅ d l = µ 0 I
v⊥=v sin θ
µ 0I v
B × 2πr = µ 0 I or B =
2π r (1)
26. (i) Refer to text on page 176 (Ampere’s circuital X
law). (1)
vII= v cos θ
(ii) According to Ampere’s circuital law, the net field is
given by B = µ 0nI
Z
(a) The net magnetic field is given by
B net = B 2 − B1 = µ 0n2 I − µ 0n1 I [Q I 2 = I 1 = I ] The path of the charged particle will be helix. As, the
charge moves linearly in the direction of the magnetic
= µ 0 I (n2 − n1 ) field with velocity v cos θ and also describe the circular
The direction is from B to A. (1) path due to velocity v sinθ. (1)
(b) As the magnetic fields due to S 1 is confined solely
inside S 1 as the solenoids are assumed to be very long. Time taken by the charge to complete one circular
So, there is no magnetic field outside S 1 due to current 2πr
in S 1 , similarly, there is no field outside S 2 . rotation, T = …(i)
v⊥
∴ B net = 0 (1)
27. (i) Refer to text on page 178 (Force on a moving charge in ⇒ f = qv ⊥ B
a uniform magnetic field). (2)
mv ⊥2
(ii) Since, force always adjusts itself in a direction which and = qv ⊥ B
becomes perpendicular to velocity, so only direction of r
velocity changes not the magnitude. Hence, the kinetic v⊥m
⇒ =r …(ii)
energy of the particle always remains constant. (1) qB
28. (i) Refer to text on page 178 (Force on a moving charge in From Eqs. (i) and (ii), we get
a uniform magnetic field). (1) 2πv ⊥ m 2πm
(ii) According to question, magnetic force on a charge F ⇒ T = =
qB ⋅ v ⊥ Bq (1)
particle is given by
F = q ( v × B) Distance moved by the particle along the magnetic field
The direction of force on the charged particle is given in one rotation (pitch of the helix path)
by ( v × B ) with the sign of charged particle, i.e. for = v || × T [Qv || = v parallel ]
α-particle, charge is positive and direction of v is + $i 2π m
$ = v cos θ ×
and direction of B is − k. Bq
So, direction of force is + ( $i × − k$ ) , i.e. + $j. 2πmv cos θ
It describes a circle with anti-clockwise motion. P=
qB (1)
For neutron
It is a neutral particle so, it goes undeviated. 30. (i) Refer to text on page 180 (Cyclotron). (2)
As F = q ( v × B) = 0 (ii) Let the mass of proton = m
For electron Charge of proton = q, mass of deuteron = 2m
Force is given by F = − e ( v × B ) Charge of deuteron = q
So, direction = − ( $i × − k$ ) ⇒ − $j Bq q
Cyclotron frequency, ν = ⇒ν∝
e − describes a circle with clockwise motion 2π m m
q
For proton frequency, ν p ∝ …(i)
m
α
q
For deuteron frequency, νd ∝ …(ii)
2m
n
From Eqs. (i) and (ii), we get, ν p = 2 νd
e
Thus, frequency of proton is twice that of deuteron.
No, both cannot be accelerated with same oscillator
(2) frequency as they have different mass. (1)
Moving Charges and Magnetism | 189

31. Refer text on page 181 (Uses of cyclotron). should be no deflection in the path of electron, then
the magnetic force should be in South direction.
32. (i) Refer to text on pages 176 and 177 (Magnetic field of a
By F = − e ( v × B ), the direction of velocity is West to
solenoid). (2)
East, the direction of force is towards South, by using
(ii) Refer to text on pages 177 and 178 (Magnetic field of a the Fleming’s left hand rule, the direction of magnetic
toriod). (1)
field (B) is perpendicularly inwards to the plane of
Magnetic field inside the open space interior of paper. (2)
the toroid Let the loop 2 be shown in the figure,
experience magnetic field B. 34. (i) Refer to text on pages 178 and 179 (Motion of a charge
in magnetic field).
No current threads the loop 2 which lie in the open
space inside the toroid. (ii) Refer to text on pages 180 (Cyclotron).
∴ By Ampere’s circuital law, 35. (i) Refer to text on pages 180 (Cyclotron). (2½)
(ii) (a) Let the mass of proton = m
∫ B⋅ d l = µ 0 (l) = 0
loop 2
Charge of proton = q
⇒ B =0 (1) Mass of α-particle = 4m
Magnetic field in the open space exterior of the Charge of α-particle = 2q
toroid Let us consider a coplanar loop 3 in the open Bq q
space of exterior of toroid. Here, each turn of toroid Cyclotron frequency, ν = ⇒ν∝
threads the loop two times in opposite directions. 2π m m
q
Therefore, net current threading the loop For proton frequency, ν p ∝
= NI − NI = 0 m
2q q
∴ By Ampere’s circuital law, For α-particle frequency, ν α ∝ or ν α ∝
4m 2m
∫ B ⋅ d l = µ 0 ( NI − NI ) = 0
loop 3 Thus, particles will not accelerate with same
⇒ B =0 cyclotron frequency. The frequency of proton is
Thus, there is no magnetic field in the open space twice than the frequency of α-particle. (1½)
interior and exterior of the toroid. (1) Bqr q
(b) Velocity, v = ⇒ v∝
33. (i) The magnetic field is in m m
constant direction from West q
East For proton velocity, v p ∝
East to West. According to B m
the question, a charged 2q
particle travels undeflected For α-particle velocity, v α ∝
4m
along a straight path with constant speed. It is only
q
possible, if the magnetic force experienced by the ⇒ vα ∝
charged particle is zero. 2m
The magnitude of magnetic force on a moving Thus, particles will not exit the dees with same
charged particle in a magnetic field is given by velocity. The velocity of proton is twice than the
F = qv B sin θ (where θ is the angle between v and B). velocity of α-particle. (1)
Here F = 0, if and only if sin θ = 0 (as v ≠ 0, 36. To find magnetic field at a point, consider Amperian
q ≠ 0, B ≠ 0). This indicates the angle between the loop around an infinitely long conducting wire as
velocity and magnetic field is 0° or 180°. shown.
Thus, the charged particle moves parallel or (i) In order to determine the magnetic field at point
anti-parallel to the magnetic field B. (2) P, which is situated at a distance a from the centre of
(ii) Yes, the final speed be equal to its initial speed as the the circular loop around the conducting wire. The
magnetic force acting on the charged particle only magnetic field (B) is tangential to the circumference
changes the direction of velocity of charged particle of the loop.
but cannot change the magnitude of velocity of
charged particle. (1) I
(iii) As, the electric field is from North
North to South, that means the + + + + + + + + P B
plate in North is positive and in a
South is negative. Thus, the East
electrons (negatively charged) Amperian loop
attract towards the positive e
As we know that from Ampere’s circuital law,
plate that means move towards
North. If we want that there ∫ B ⋅dl = µ 0 I
– – – – – – – –
Electric field
South
190 |

As ∫ dl = 2πa 39. Magnetic field, B = 100 G = 100 × 10−4 T = 10−2 T


So, B 2π a = µ 0 I To design the solenoid, let us find the product of current
µ I 2 and number of turns in the solenoid.
B= 0 ×
2π a 2 The magnitude of magnetic field, B = µ 0nI (1)
−2
µ 0 2I B 10
⇒ B= or nI = = . × 10−7
× 314
4π a µ0 4
The direction of magnetic field will be determined by ⇒ nI = 7961 ≈ 8000
right hand thumb rule. (1½)
Here, the product of nI is 8000. (1)
(ii)
P Current, I = 8 A and number of turns, n = 1000
The other design is I = 10 A and n = 800/m.
3A
Electron This is the most appropriate design as per the
0.6 m requirement. (1)
40. Refer to example 4 on page 178.
(i) 0 (ii) 2.45 × 10−2 T (iii) 0
Q 41. Here, energy of electron, E ′ = 2000 eV
From the given figure, if we have to calculate force on . ×10−19 J = 3.2 × 10−16 J
E ′ = 2000 × 16
a long straight conductor.
1
Force acting on the electron, Now, B = 0.2 T, r = ?, E ′ = mv 2
2
F = qvB
2E ′
Due to wire, the field created is ⇒ v =
µ I µ0 × 3 m
B= 0 = mv 2
2πR 2 × π × 06 . Bev =
r (1)
4 π × 10− 7 × 3
= = 10− 6 T mv m 2E ′ 2E ′m
2 × π × 06. ∴ r= = =
Be Be m Be
Force, F = Bqv
2 × 3.2 × 10−16 × 9 × 10−31
. × 10− 19 × 2 × 107 × 10− 6
Now, F = 16 =
0.2 × 1.6 × 10−19
16
= × 10− 19 × 2 × 107 × 10− 6
10 = 7.5 × 10−4 m (1)
= 16 × 2 × 10− 19 + 6 − 6 42. As the path of an electron remains undeflected, hence
F = 32 × 10− 19 N net force on the electron must be zero (balanced).
(1½)
Consider the diagram, where an electron beam enters
100
37. Here, I = 2.5 A,n = = 200 the region in which magnetic field ( B ) is perpendicularly
0. 50 inward ⊗ and electric field ( E ) is downward.
(i) B = µ 0nI = 4 π × 10−7 × 200 × 25
. + + + + +
−4
B = 6. 28 × 10 T (1)
E v
µ nI 4 π × 10−7 × 200 × 2.5
(ii) B = 0 = = 3.14 × 10−4 T e
2 2 (1)
– – – – – B
38. Number of turns, N = 850 × 5, l = 1 m, I = 5 A
2 Force on electron due to electric field = eE [upward]
22 2 3 −2  2 Force on electron due to magnetic field = evB
Area of cross-section, A = πr =  × 10  m
7 2  [downward]
Magnetic field at the centre of solenoid, B = µ 0 NI /l [where, v = speed, e = electronic charge] (1)
= 4π × 10−7 × (850 × 5) × 5/1 For no deflection electric force will must be balanced by
= 2.671 × 10−2 T magnetic force
(1)
eE = evB
∴ Magnetic flux = BA 4
2 E 3.4 × 104 given, E = 3.4 × 10 V / m
22  3  ⇒ v= =  
= 2.671 × 10−2 × ×  × 10−2  B 2 × 10 −3 −3
B = 2 × 10 T 
7 2 
= 1.89 × 10−5 Wb = 1.7 × 107 m / s (1)
(1)
Moving Charges and Magnetism | 191

When electric field is removed, only magnetic field B = magnitude of the magnetic field
remains, hence path will be circular as shown in the and θ = angle between v and B.
figure. Consider the diagram shown below.
v
v Y
e v
r B

O X
v B
v
mv Let the horizontal direction is X and vertical direction
Radius of the circular path (r ) is given by r = is Y.
eB (1)
F = q ( v × B) = −e (v × B)
where, m is mass of the electron. [along Z -axis which is perpendicular
B is magnitude of the magnetic field to the plane of paper]
mv 9.1 × 10−31 × 1.7 × 107 Hence, direction of force will be perpendicularly
⇒ r= =
eB 1.6 × 10−19 × 2 × 10−3 outward.
= 4.83 × 10−31 + 29 | F | = evB [Qθ = 90° ]
= 4.83 × 10−2 m = 4.83 cm ≈ 5 cm (1)
= 1.6 × 10−19 × 2 × 106 × 0.32 × 10−4
= 1.024 × 10−17 N (2)
43. According to the question,
45. Frequency of alternating electric field in cyclotron is
q = 1.6 × 10−19 C
given by
v = 5 × 107 m / s qB
f =
θ = 30° , B = 1 Wb m −2 2πm (1/2)

F = qvB sinθ Here, q = 3. 2 × 10−19 C,


= 1.6 × 10−19 × 5 × 107 × 1 × sin 30° π
. × 10−27 kg and B =
m = 67 T
= 4 × 10 −12 N 2
(2)
q ( π / 2)
44. The force on a charged particle moving in a magnetic ∴ f =
field B with velocity v and having charge q is q ( v × B ). 2π m
The magnitude of force is (3.2 × 10−19 ) × (( π / 2)
=
| F | = q | v | | B |sin θ 2 × ( π × 6.7 × 10−27 (1)
7
where, q = charge, = 1. 2 × 10 cycle/s (1/2)
v = speed of the charged particle,

TOPIC Magnetic Force and Torque


03 Experienced by a Current Loop
Force on a Current Carrying Conductor Let us consider a portion of length l and cross-sectional
area A of a straight conductor carrying a current I .
in a Uniform Magnetic Field
Let the magnetic field B be in the plane of the paper
When a current carrying conductor is placed in a directed upwards and making an angle θ with the
uniform magnetic field, then due to motion of free direction of current.
electrons inside the conductor, a magnetic force acts on
Each electron of conductor experiences a magnetic
it.
Lorentz force, which is given by
B
F = e ( v d × B) ...(i)
θ If n be the number of free electrons per unit volume in the
X′ X
l vd conductor, then total number of electrons in the conductor is
e e
N = n × volume of conductor
Current carrying conductor in a uniform magnetic field N = nAl [A → Area of cross section]
192 |

∴ Total force on the conductor is Example 1 A 3.0 cm wire carrying a current of 10 A


F = N ⋅F is placed inside a solenoid perpendicular to its axis. The
= nAl [ −e ( v d × B)] = enA [ −l v d × B ] magnetic field inside the solenoid is given to be 0.27 T.
= enA [ v d ⋅ l × B ] [ −l v d = v d l] What is the magnetic force on the wire?
r r Sol. Magnetic force on the wire,
= neAv d [ l × B ] = I [ l × B ] [Q I = neAv d ]
F = BIl sinθ = BIl sin90°
Magnitude of force is = 0.27 × 10 × 3 × 10−2 [Qθ = 90°]
F = I B l sin θ = 81 −2
. × 10 N
or F = I ( l × B) Example 2 A straight wire of mass 200 g and length
If θ = 0° or 180°, then F = I Bl sin 0° = 0 1.5 m carries a current of 4 A. It is suspended in mid air
[Qsin 0° = 0 and sin 180° = 0] by a uniform horizontal magnetic field B. What is the
magnitude of the magnetic field?
It means a conductor placed parallel to direction of
magnetic field, experiences no force due to magnetic l
field. mg
If θ = 90°, then force is maximum. B

F max = IBl sin 90° = IBl [Qsin 90° = 1]


It means a conductor placed perpendicular to direction of Sol. Applying Fleming’s rule, we find that upward force F of
magnetic field, experiences maximum force. magnitude IlB acts. For mid air suspension this must be
balanced by the force due to gravity.
mg
Rules to Find Out the ∴ mg = IlB ⇒ B =
Il
Direction of Force Given, m = 200 g = 0.2 kg, g = 9.8 ms −2 , I = 4 A, l = 15
. m
The direction of the force acting on a current carrying 0.2 × 9.8
We have, B = = 0 .327 T
conductor in a magnetic field can be found by any of the 4 × 1.5
following two rules.
(i) Right Hand Palm Rule If we stretch our right Force between Two Parallel Current
hand palm such that the thumb points in the Carrying Conductors
direction of the current (I) and the stretched fingers To find the force on a current carrying wire due to a
in the direction of the magnetic field B, then the second current carrying wire, first find the magnetic field
force F on the conductor will be perpendicular to due to second wire at the site of first wire. Then, find the
the palm in the direction of pushing by the palm as force on the first wire due to that field.
shown in the Fig. (a). Let us consider A1 B1 and A 2 B 2 are B1 r B2
F I F two infinite long straight B1
B B2
B
conductors.
P
I 1 and I 2 are the currents flowing F1 F2
through them and these are at r I 1 I2
I
distance apart. A1
(b) A2
(a)
Magnetic field induction at a point Two parallel
(ii) Fleming’s Left Hand Rule If the fore-finger, the P on conductor A 2 B 2 due to current carrying
middle-finger and the thumb of the left hand are current
stretched mutually at right angles to one another µ 2I
I 1 passing through A1 B1 is B1 = 0 1
such that the fore-finger points in the direction of 4 πr
the magnetic field B and the middle-finger in the
direction of the current I, then the thumb will point The unit length of A 2 B 2 will experience a force as
in the direction of the force F on the conductor as F 2 = B1 I 2 × l = B1 I 2l
shown in the Fig. (b). µ 2I I
or F2 = 0 ⋅ 1 2 ⋅ l
4π r
Moving Charges and Magnetism | 193

Conductor A1 B1 also experiences the same amount of Torque Experienced by a Current Loop in
force, directed towards the wire A 2 B 2 . Uniform Magnetic Field (Magnetic Dipole)
Therefore, force between two current carrying parallel Consider a rectangular loop ABCD be suspended in a
conductors per unit length is uniform magnetic field B. Let AB = CD = b and
µ 0 2I 1I 2 AD = BC = a . Let I be the current flowing through the loop.
F = ⋅ a
4π r Rotation axis
B ω
C
Two linear parallel conductors carrying currents in the b I
N B
same direction attract each other while carrying currents I S
τ b
in opposite direction they repel each other.
A
M D
Definition of Ampere
I
(In terms of the force) I
+−
One ampere is the current which flows through each of
the two parallel uniform long linear conductors, which A rectangular current carrying coil in uniform magnetic field
are placed in free space at a distance of 1 m from each
other and which attract or repel each other with a force Case I The rectangular loop is placed such that the
of 2 × 10 −7 N/m of their lengths. uniform magnetic field B is in the plane of loop.
No force is exerted by the magnetic field on the arms AD
Example 3 Calculate the force per unit length on a and BC (Q they are parallel to the magnetic field).
long straight wire carrying current of 4 A due to a Magnetic field exerts a force F1 on arm AB,
parallel wire carrying 6 A current, if the distance ∴ F1 = IbB
between the wires is 3 cm.
Magnetic field exerts a force F 2 on arm CD,
Sol. Given, I 1 = 4 A, I 2 = 6 A, r = 3 cm = 0.03 m
F 2 = IbB = F1
µ 0 2 I1 I 2
F = ⋅ F1 and F 2 are equal and opposite, so net force on the
4π r
−7
10 × 2 × 4 × 6
loop is zero. But line of action of F1 and F 2 are opposite
= and parallel, so they form a couple.
0.03
F2
= 1.6 × 10− 4 N/m

Example 4 A short conductor of length 5 cm is


a/2 a/2
placed parallel to a long conductor of length 1.5 m near
its centre. The conductors carry currents 4 A and 3 A
respectively in the same direction. What is the total m
force experienced by the long conductor when they are
3 cm apart? F1

Sol. Force on long conductor is equal and opposite to the The torque produced due to couple on the loop rotates
µ 2I I l the loop in anti-clockwise direction.
force on small conductor = 0 ⋅ 1 2
4π r Torque, τ = r × F
Given, I 1 = 4 A, a a
I 2 = 3 A, So, τ = F1 + F 2 [Qsin 90° = 1]
2 2
r = 3 × 10− 2 m, (Q Torque = Force × Perpendicular distance
l = 5 × 10− 2 m of line of action)
4 π × 10− 7 × 2 × 4 × 3 × 5 × 10− 2 a a
⇒ F = ⇒ τ = IbB + IbB = I ( ab) B = IAB
4 π × 3 × 10− 2 2 2
= 4 × 10− 6 N where, b be breadth of the rectangular coil, a be length
of the rectangular coil and A = ab (area of the coil).
194 |

Case II The plane of the loop is not along the magnetic The presence of this torque is also the reason why a small
field, but makes an angle with it. magnet or any magnetic dipole aligns itself with the external
C magnetic field.
I
B
I
B
Example 5 A circular coil of 20 turns and radius
N I m θD S 10 cm carries a current of 5 A. It is placed in a uniform
magnetic field of 0.10 T. Find the torque acting on the
A
coil, when the magnetic field is applied in the plane of
The area vector of the loop ABCD makes an coil.
arbitrary angle θ with the magnetic field Sol. Given, total number of turns, N = 20
Angle between the field and the normal to the coil is θ. Radius, r = 10 cm = 10 × 10− 2 m
Forces on BC and DA are equal and opposite and they Current, I = 5 A; Angle, θ = 90 °
cancel each other as they are collinear. External uniform magnetic field ( B ) = 0.10 T
Force on AB is F1 and force on CD is F 2 . Torque, τ = ?
As, torque, τ = BINA sin θ
and F1 = F 2 = IbB ⇒ = 0.10 × 5 × 20 × 0.0314 × sin 90° = 0.314 N-m
Magnitude of torque on the loop is as shown in figure
below: Example 6 Calculate the torque of a 100 turns
F2
rectangular coil of length 40 cm and breadth 20 cm,
carrying a current of 10 A, when placed making an
a/2 angle of 60° with a magnetic field of 5 T.
a/2 sin θ
B
θ Sol. Given, I = 10 A, N = 100, l = 40 cm , b = 20 cm
a/2 m B = 5 T, θ = 60 °
A = l × b = 40 × 20 = 800 cm 2 = 8 × 10 −2 m 2
F1 Q τ = NBIA sin θ = 100 × 5 × 10 × 8 × 10 −2 × sin 60°
Top view of the loop. The forces F1 and F2 acting
on the area AB and CD are indicated
= 346.41 N-m

a a Example 7 Find the magnitude C


I
τ = F1 sin θ + F 2 sin θ
2 2 of magnetic moment of the current D
carrying loop ABCDEFA. Each side of B
τ = IabB sin θ = IAB sin θ
the loop is 10 cm long and current
where, A = ab (area of coil) in the loop is I = 2.0 A. A
E

The torque on the loop can be expressed as the vector Sol. By assuming two equal and
product of the magnetic moment of the coil and the opposite currents in BE, two F
magnetic field current carrying loops (ABEFA and BCDEB) are formed.
Their magnetic moments are equal in magnitude but
τ = MB sin θn$ = M × B perpendicular to each other. Hence,
where, M = NI A is magnetic moment of the loop, its M net = M 2 + M 2 = 2M
unit is ampere-metre 2 . Q M = IA = 2 × (l × b ) = 2 × (0.1) (0.1) = 0.02 A-m 2
This is analogous to the electrostatic case (electric dipole
∴ M net = ( 2) (0.02) = 0.028 A-m 2
of dipole moment p in an electric field E )
When M and B are parallel, then current loop is in stable
equilibrium. Any small rotation of the loop produces a Circular Current Loop as a
torque which brings it back to its original position. When Magnetic Dipole
M and B are anti-parallel, then current loop is in
unstable equilibrium. A current loop behaves as a magnetic dipole. If we look
at the upper face, current is anti-clockwise, so it has
Magnetic moment of the loop of N turns, M = NIA North polarity. If we look at lower face, current is
The total torque on the coil is given by clockwise, so it has South polarity.
τ = NIAB sin θ = ( NIA) B sin θ That means current loop behaves as a system of two
equal and opposite magnetic poles hence, it acts as a
τ = NIAB sin θ magnetic dipole.
Moving Charges and Magnetism | 195

Magnetic dipole moment of loop, M = NIA where, h = Planck’s constant and n is natural number
where, I = current flowing M i.e. n = 1, 2, 3,...
through the loop, n Upper face e nh e
⇒ M = ⋅ = nh
A = area enclosed by the loop 2m e 2π 4πm e
and N = number of turns in the coil.
For n = 1, M will be minimum.
The magnitude of magnetic field
on the axis of a circular loop of eh
∴ M min =
radius R, carrying steady current I 4πm e
Lower face
µ 0IR 2 It is Bohr’s magneton, which is defined as magnetic
is given by B =
2( x 2 + R 2 ) 3/ 2 moment of revolving electron in its first orbit. Its value is
For x >> R , 9.27 × 10 −24 A - m 2 .
µ IR 2 µ IA Example 8
B= 0 3 = 0 3 [Q A = πR 2 ] An electron in a hydrogen atom is
2x 2 πx moving with a speed of 2.3 × 106 ms − 1 in an orbit of
µ 0M radius 0.53 Å. Calculate the magnetic moment of the
⇒ B= [Q M = IA] revolving electron.
2 πx 3
Sol. Given, v = 2.3 × 106 ms −1 , r = 0.53 Å = 0.53 × 10− 10 m
e e ev
Magnetic Dipole Moment of a Equivalent current, I =
T
=
2π r
=
2πr
Revolving Electron v
1.6 × 10− 19 × 2.3 × 106
An electron being a charged particle, constitutes a current = = 1.105 × 10− 3 A
2 × 3.14 × 0.53 × 10− 10
while moving in its circular orbit around the nucleus
∴ Magnetic moment,
(Q Moving charge constitutes a current as well as
M = IA = I ( πr 2 ) = 1.105 × 10− 3 × 3.14 × (0.53 × 10− 10 )2
magnetic field). If T is the time period of revolution,
e = 9.75 × 10− 24 A-m 2
then current constituted by electron is I = … (i)
T
where, e = charge of electron. Moving Coil Galvanometer
If r is the orbital radius of electron and its orbital speed is It is an easy and sensitive instrument to defect and
v, then measure electric field.
2πr e ev
T = ⇒ I = = [from Eq. (i)] Principle
v 2πr 2πr
v Its working is based on the fact that when a current
Magnetic moment of revolving electron, M = IA carrying coil is placed in a magnetic field, it experiences a
ev evr torque.
M = πr 2 = H
2 πr 2
The direction of this magnetic moment is into the plane Phosphor
bronze
of the paper. strip
e e −e l T1 T2
M = (m e vr ) = l or M = M
2m e 2m e 2me P
S
where, l = m e vr is angular momentum of the electron.
M e L
= is a constant, called gyromagnetic ratio, its N S
l 2m e
value is 8 .8 × 10 10 C/kg for an electron. Q R
S′
From Bohr’s hypothesis, angular momentum can have Core
only some discrete values,
nh Schematic arrangement of moving coil galvanometer
l=

196 |

Note Dead beat galvanometer is one in which the coil comes to


Construction rest at once after the passage of current through it. The deflection
The moving coil galvanometer consists of a coil with can be noted in no time.
many turns free to rotate about a fixed axis, in a uniform Example 9
radial magnetic field. There is a cylindrical soft iron core In order to increase the current
which not only makes the field radial but also increases sensitivity of a moving coil galvanometer by 50%, its
the strength of the magnetic field. When a current flows resistance is increased so that the new resistance
through the coil, a torque acts on it. becomes twice its initial resistance. By what factor does
its voltage sensitivity change?
Working Sol. Increased current sensitivity, I S′ = I S +
50 I S 150 I S
=
Suppose the coil PQRS is suspended freely in the 100 100
3 IS
magnetic field. Let l be length PQ or RS of the coil, = , R ′ = 2R
b be breadth QR or SP of the coil, N be number of turns 2
IS
in the coil and area of each turn of the coil, A = l × b.Let Initial voltage sensitivity, VS = ...(i)
B be strength of the magnetic field in which coil is R
3
suspended and I is current passing through the coil in IS
I S′
the direction PQRS. New voltage sensitivity, V S′ = = 2
R′ 2R
Let at any instant, α be the angle, which normal drawn 3
on the plane of the coil makes with the direction of ⇒ V ′S = VS [from Eq. (i)]
4
magnetic field. The rectangular coil carrying current V − V S′
when placed in the magnetic field experiences a torque ∴ % decrease in voltage sensitivity = S × 100
VS
whose magnitude is given by τ = NIBA sin α .Due to
 3
deflecting torque, the coil rotates and suspension wire = 1 −  × 100 = 25%
 4
gets twisted. A restoring torque is set up in the
suspension wire. Note This topic has been frequently asked in the previous year
Let θ be the twist produced in the phosphor bronze strip exams, i.e. in year 2016, 2015, 2014, 2010, 2009.
due to rotation of the coil and k be the restoring torque
per unit twist of the phosphor bronze strip. Then, Conversion of a Galvanometer
Total restoring torque produced = k θ into Ammeter
In equilibrium position of the coil, Deflecting torque = To convert a galvanometer into ammeter, its resistance
Restoring torque needs to be lowered, so that maximum current can pass
k k through it and it can give exact reading.
NIBA = k θ ⇒ I = θ = G θ where, =G Ammeter
NBA NBA
It is known as galvanometer constant. S
i.e. θ ∝ I . It means that the deflection produced is
proportional to the current flowing through the
(I – Ig ) (I – Ig )
galvanometer.
Current sensitivity of the galvanometer is the deflection G
per unit current flowing through it. I Ig Ig I
θ NAB
It is given by I S = = A shunt (low resistance) is connected in parallel with the
I k
galvanometer.
Its unit is rad/A or div/A.
Voltage sensitivity is the deflection per unit voltage.  Ig 
S =  G

It is given by  I − Ig 
θ  NAB  I NAB I NAB where, I = total current in circuit,
VS = =   or VS = × =
V  k V k IR kR G = resistance of the galvanometer,
[Q according to Ohm’s law, V = IR ] S = resistance of the shunt (low resistance)
Its unit is rad/V or div/V. and I g = current through the galvanometer.
Moving Charges and Magnetism | 197

Example 10 A galvanometer of resistance 15 Ω gives


full scale deflection for a current of 2 mA. Calculate the
Topic Practice 3
shunt resistance needed to convert it to an ammeter of
range 0 to 5 A.
Multiple Choice Questions
Sol. Given, G = 15 Ω, Ig = 2 mA = 2 × 10−3 A , I = 5 A
1 Mark
Ig G
∴ Shunt resistance, S =
I − Ig 1. Two similar coils are placed mutually
−3 perpendicular such that their centres coincide.
2 × 10 × 15
= = 0.006 Ω At centre, the ratio of the magnetic field due to
5 − 2 × 10−3 one coil and the resultant magnetic field of both
This resistance S = 0.006 Ω is connected in parallel with the coils for same current will be
galvanometer. The small resistance is connected in parallel,
(a) 1 : 2 (b) 1 : 2 (c) 2 : 1 (d) 3 :1
because we have to decrease the resistance of the galvanometer
so that most of the current passes through it and it gives the 2. Magnetic moment (or magnetic dipole
exact value of the current. moment) of a current-carrying coil is given by
I
Conversion of a Galvanometer into (a) m = IN A (b) m =
A
Voltmeter (c) m = N A (d) m =
NI
To convert a galvanometer into voltmeter, its resistance A
needs to be increased, so that there is no potential drop 3. A moving coil galvanometer is an instrument
across it because with high resistance no current passes which
through it. (a) is used to measure EMF
(b) is used to measure potential difference
(c) is used to measure resistance
R (d) is a deflection instrument which gives a
A G B
Ig deflection when a current flows through its coil
V
4. In a moving coil galvanometer of coil of N turns
Voltmeter of area A have a spring of stiffness k. If coil is
deflected by some angle φ due to flow of I
A high resistance is connected in series with the
galvanometer, then the value of R is given by current in uniform radial magnetic field B, then
NAB  k 
V (a) φ =  I (b) φ =  I
R= −G  k   BNA 
Ig
kA  BN 
(c) φ =  I (d) φ =  I
where, V = potential difference across the terminals A and B  BN   kA 
Ig = current through the galvanometer 5. To make the field radial in a moving coil
(full scale deflection current), galvanometer,
R = high resistance (a) number of turns of coil is kept small
and G = resistance of the galvanometer. (b) magnet is taken in the form of horse-shoe
(c) poles are of very strong magnets
Example 11 The full scale deflection current of a (d) poles are cylindrically cut
galvanometer of resistance 1 Ω is 5 mA. How will you
convert it into a voltmeter of range 5 V? Very Short Answer Type Questions
Sol. From the relation, V = I g (G + R ), we have 1 Mark
V  5 
R= –G= –3
 –1 6. A coil of N turns and radius R carries a current I.
Ig  5 × 10  It is unwound and rewound to make another
= 999 Ω coil of radius R / 2 having same number of turns
i.e., a resistance of 999 Ω should be connected in series with N . Keeping the current I same, find the ratio of
the galvanometer to convert it into a voltmeter of desired the magnetic moments of the other new coil
range. and the circular coil.
198 |

7. A conducting loop carrying a current I is placed current I 1 is along the X-axis. The other carrying
in a uniform magnetic field, pointing into the current I 2 is along a line parallel to Y -axis, given by
plane of the paper as shown in the figure, then x = 0 and z = d . Find the force exerted at point O 2
the loop will have a tendency to expand. Explain. because of the wire along the X-axis.
Y Z
O2
I I2
X
d

Y
O1
8. Give the magnitude of torque which acts on a I1
coil carrying current placed in a uniform radial
X NCERT Exemplar
magnetic field.
20. (i) Two long straight parallel conductors a and
9. Write the underlying principle of a moving coil
b carrying steady currents I a and Ib
galvanometer.
respectively are separated by a distance d.
10. Why should the spring/suspension wire in a Write the magnitude and direction, what is
moving coil galvanometer have low torsional the nature and magnitude of the force
constant? between the two conductors?
(ii) Show with the help of a diagram, how the
11. Why is a coil wrapped on a conducting frame in
force between the two conductors would
a galvanometer?
change when the currents in them flow in
12. State how a moving coil galvanometer can be the opposite directions.
converted into an ammeter. ISC 2016
21. Figure below shows two infinitely long and thin
13. In a moving coil galvanometer, what is meant current carrying conductors X and Y kept in
by a radial magnetic field? ISC 2012 vacuum, parallel to each other at a distance a.
I1
14. State two factors on which the sensitivity of a X
moving coil galvanometer depends. ISC 2006

15. What is the purpose of concave pole pieces in a a

moving coil galvanometer? ISC 2001 I2


Y
16. The coils in certain galvanometers, have a fixed
core made of a non-magnetic metallic material. (i ) How much force per unit length act on the
Why does the oscillating coil come to rest so, conductor Y due to the current flowing
quickly in such a core? through X? Write your answer in terms of
 µ0 
17. A voltmeter, an ammeter and a resistance are   , I 1, I 2 and a.
 4π 
connected in series with a lead accumulator.
The voltmeter gives some deflection but the [Derivation of formula is not required]
deflection of ammeter is zero. Comment. (ii ) Define ampere in terms of force between
two current carrying conductors.
Short Answer Type I Questions ISC 2016, 13, 11, 08

2 Marks 22. A rectangular coil of sides l and b carrying a


current I is subjected to a uniform magnetic
18. Two long parallel wires carrying a current I, field B acting perpendicular to its plane. Obtain
separated by a distance r are exerting a force F the expression for the torque acting on it.
on each other. If the distance between them is
23. A circular coil of closely wound N turns and
increased to 2r and current in each wire is
radius r carries a current I. Write the
reduced from I to I/2, then what will be the
expressions for the following:
force between them?
(i) The magnetic field at its centre.
19. Two long wires carrying currents I 1 and I 2 are
(ii) The magnetic moment of this coil.
arranged as shown in the figure. One carrying
Moving Charges and Magnetism | 199

24. Define current sensitivity and voltage (i ) What is the effect of each of the magnetic
sensitivity of galvanometer. Increasing the fields B F and B H on the needle?
current sensitivity may not necessarily increase (ii ) When the needle is in equilibrium, obtain an
the voltage sensitivity of a galvanometer, justify expression for angle θ made by the needle
your answer. with B H in terms of B F and B H only. ISC 2010
25. How is a moving coil galvanometer converted 30. Obtain the formula, I = kθ for a moving coil
into a voltmeter? Explain giving the galvanometer given the deflecting torque
necessary circuit diagram and the required τ = M × B, where M is the magnetic dipole
mathematical relation used. moment of the coil placed in the magnetic field
B, I is the current in the galvanometer and θ is
Short Answer Type II Questions the deflection. ISC 2008

3 Marks 31. An electron of mass me revolves around a


nucleus of charge +Ze. Show that it behaves like
26. Draw a labelled diagram of a moving coil a tiny magnetic dipole. Hence, prove that the
galvanometer and explain its working. What is magnetic moment associated with it is
the function of radial magnetic field inside e
the coil? expressed as µ = − L, where L is the orbital
2me
27. A uniform conducting wire of length 12 a and angular momentum of the electron. Give the
resistance R is wound up as a current carrying significance of negative sign.
coil in the shape of
(i) an equilateral triangle of side a,
32. (i) Two long, parallel conductors carrying
currents I 1 and I 2 in the same direction.
(ii) a square of sides a and Deduce an expression for the force per
(iii) a regular hexagon of side a. The coil is unit length acting on one of the conductors
connected to a voltage source V0. Find the due to the other. Is this force attractive or
magnetic moment of the coils in each case. repulsive?
NCERT Exemplar
(ii) Find the expression for magnetic dipole
28. Two very long conductors 1 2
moment of a revolving electron. What is
carrying currents I 1 and I 2 are d Bohr magneton?
separated by a distance d as I1 I2
shown in the figure. Give the r 33. Answers the following questions.
expressions (do not derive) for (i) Write two reasons why a galvanometer
the magnetic fields B 12 and B 21 cannot be used as such to measure the
P Q
at P and Q respectively, current in a given circuit. Name any two
produced by the electric factors on which the current sensitivity of
currents I 2 and I 1 . What are the directions of a galvanometer depends.
these fields? (ii) Why is it necessary to introduce a
Define the SI unit of currents using the below cylindrical soft iron core inside the coil of a
set up. ISC 2007 galvanometer?

29. A small magnetic needle NS having magnetic 34. State the principle of working of a
dipole moment p m is kept in two uniform and galvanometer. A galvanometer of resistance G
perpendicular magnetic fields B F and B H as is converted into a voltmeter to measure upto V
shown below. volts by connecting a resistance R 1 in series
North with the coil. If a resistance R 2 is connected in
BH
series with it, then it can measure upto V /2
θ N volts. Find the resistance, in terms of R 1 and R 2,
West BF required to be connected to convert it into a
S East voltmeter that can read upto 2V. Also, find the
resistance G of the galvanometer in terms of R 1
South and R 2.
200 |

Long Answer Type Questions 40. A uniform magnetic field of 1.5 T exists in a
5 Marks cylindrical region of radius 10.0 cm, its
direction parallel to the axis along East to West.
35. Explain using a labelled diagram, the principle A wire carrying current of 7.0 A in the North to
and working of a moving coil galvanometer. South direction passes through this region.
What is the function of What is the magnitude and direction of the
(i) uniform radial magnetic field? force on the wire, if
(ii) soft iron core? Also, define the terms. (i) the wire intersect the axis?
(iii) current sensitivity? (ii) the wire is turned from North-South to
North East-North West direction?
(iv) voltage sensitivity of a galvanometer?
(iii) the wire in the North-South direction is
Why does increasing the current sensitivity
lowered from the axis by a distance of
not necessarily increase voltage
6.0 cm? NCERT (3 M)
sensitivity?
36. (i) Explain giving reasons, the basic difference 41. A solenoid 60 cm long and of radius 4.0 cm has
in converting a galvanometer into 3 layers of winding of 300 turns each. A 2.0 cm
long wire of mass 2.5 g lies inside the solenoid
(a) a voltmeter.
(near its centre) normal to its axis, both the wire
(b) an ammeter. and the axis of the solenoid are in the
(ii) Two long straight parallel conductors horizontal plane. The wire is connected through
carrying steady currents I 1 and I 2 are two leads parallel to the axis of the solenoid to
separated by a distance d. Explain briefly, an external battery which supplies a current of
with the help of a suitable diagram, how 6.0 A in the wire. What value of current (with
the magnetic field due to one conductor appropriate sense of circulation) in the windings
acts on the other. Hence, deduce the of the solenoid can support the weight of the
expression for the force acting between wire? (g = 9.8 m/s 2) NCERT (3 M)
the two conductors. Mention the nature of
this force. 42. A conductor of length 2 m carrying current of
2 A is held parallel to an infinitely long
Numerical Problems conductor carrying current of 10 A at a distance
of 100 mm. Find the force on a small conductor.
37. What is the magnitude of magnetic force per
(2 M)
unit length on a wire carrying a current of 8 A
making an angle of 30° with the direction of a 43. Two long and parallel straight wires A and B
uniform magnetic field of 0.15 T? NCERT (2 M) carrying currents of 8.0 A and 5.0 A in the same
direction are separated by a distance of 4.0 cm.
38. A long straight wire carrying current of 25 A
Estimate the force on a 10 cm section of wire A.
rests on a table shown in the figure. Another NCERT (2 M)
wire PQ of length 1 m , mass 2 . 5 g carries the
same current but in the opposite direction. The 44. A wire AB is carrying a steady current of 12 A
wire PQ is free to slide up and down. To what and is lying on the table. Another wire CD
height will PQ rise? NCERT Exemplar (3 M) carrying 5 A is held directly above AB at a
height of 1 mm.
39. A 3.0 cm wire carrying a current of 10 A is Find the mass per unit length of the wire CD, so
placed inside a solenoid perpendicular to its that it remains suspended at its position, when
axis. The magnetic field inside the solenoid is left free. Give the direction of the current
given to be 0.27 T. What is the magnetic force flowing in CD with respect to that in AB. (Take
on the wire? NCERT (3 M) the value of g = 10 ms −2) (3 M)
Moving Charges and Magnetism | 201

45. A circular coil of 100 turns, radius 10 cm carries 51. A rectangular coil of area 2 × 10 − 4 m 2 and
a current of 5 A. It is suspended vertically in a 40 turns is pivoted about one of its vertical sides.
uniform magnetic field of 0 . 5 T, the field lines The coil is in a radial horizontal field of 60 G .
making an angle of 60° with the plane of the coil. What is the torsional constant of the hair springs
Calculate the magnitude of the torque that must connected to the coil, if a current of 4.0 mA
be applied to it to prevent it from turning. (2 M) produces an angular deflection of 16°? (2 M)

46. A circular coil of N turns and radius R carries a 52. Two moving coil meters M 1 and M 2 having the
current I. It is unwound and rewound to make following particulars
another coil of radius R/2, current I remaining
R 1 = 10 Ω, N 1 = 30 , A1 = 3.6 × 10 − 3 m 2,
the same. Calculate the ratio of the magnetic
moments of the new coil and the original coil. B 1 = 0.25 T
ISC 2003 (2 M) R 2 = 14 Ω, N 2 = 42 , A2 = 1 .8 × 10 − 3 m 2,
47. A square coil of side 10 cm consists of 20 turns B 2 = 0.50 T (The spring constants are identical
and carries current of 12 A . The coil is for the two meters). Determine the ratio of (i)
suspended vertically and normal to the plane of current sensitivity and (ii) voltage sensitivity of
the coil makes an angle of 30° with the M 2 and M 1 . NCERT (3 M)
direction of a uniform horizontal magnetic field
of magnitude 0.80 T. What is the magnitude of
53. A galvanometer coil has a resistance of 15 Ω
and the meter shows full scale deflection for a
torque experienced by the coil? NCERT (2 M)
current of 4 mA. How will you convert the meter
48. (i) A circular coil of 30 turns and radius 8.0 into an ammeter of range 0 to 6 A? NCERT (2 M)
cm carrying a current of 6.0 A is suspended
54. When a galvanometer having 30 division scale
vertically in a uniform horizontal magnetic and 100 Ω resistance is connected in series to
field of magnitude 1.0 T. The field lines the battery of emf 3 V through a resistance of
make an angle 60° with the normal of the 200 Ω, shows full scale deflection. Find the
coil. Calculate the magnitude of the figure of merit of the galvanometer in
counter torque that must be applied to microampere. (2 M)
prevent the coil from turning.
55. A galvanometer coil has a resistance of 12 Ω and
(ii) Would your answer change, if the circular the meter shows full scale deflection for a
coil were replaced by a planar coil of some current of 3 mA. How will you convert the
irregular shape that encloses the same meter into a voltmeter of range 0 to 18 V?
area? All other particulars are also
NCERT (2 M)
unaltered. NCERT (3 M)

49. A circular coil of 20 turns and radius 10 cm is Hints & Solutions


placed in a uniform magnetic field of 0.1 T 1. (a) I
normal to the plane of the coil. If the current in
the coil is 5.0 A, what is the (i) total torque on
the coil, (ii) total force on the coil (iii) average
force on each electron in the coil due to the
magnetic field? For a single coil, magnetic field,
(The coil is made of copper wire of µ I
Bs = 0 …(i)
cross-sectional area 10 −5 m 2 and the free 2R
electron density in copper is given to be
I
about 10 29 /m3 ). NCERT (3 M)
50. The electron in a H-atom circles around the
proton with a speed of 2.18 × 106 ms −1 in an I
orbit of radius 5. 3 × 10 −11 m. For two coils magnetic field,
Calculate µ 0I
B T = B12 + B 22 = 2 …(ii)
(i) the equivalent current. 2R
(ii) magnetic field produced at the proton. On comparing Eqs. (i) and (ii), we get
Given, charge on electron is 1.6 × 10 −19 C BS 1
=
and µ 0 = 4 π × 10 −7 T mA −1 . (2 M) BT 2
202 |

2. (a) Magnetic moment of a current-carrying loop is A


m = NIA
S
where, N is number of turns, I is current flowing on a
loop and A is area of cross-section of loop.
3. (d) A moving coil galvanometer is a sensitive instrument (I – Ig ) (I – Ig )
which is used to measure a deflection when a current
flows through its coil.
G
4. (a) When a current flows through the coil, a torque acts I Ig Ig I
on it. This torque is given by τ = NIAB .
where, the symbols have their usual meaning. Since, the 13. Radial magnetic field When plane of the coil is
field is radial by design, we have taken sin θ = 1 in the parallel to the magnetic field such that the area vector of
above expression for the torque. The magnetic torque the coil is perpendicular to the magnetic field, the field
NIAB tends to rotate the coil. is called radial magnetic field.
A spring S P provides a counter torque kφ that balances A
the magnetic torque NIAB; resulting in a steady angular
deflection φ. In equilibrium kφ = NIAB .
where, k is the torsional constant of the spring, i.e. the B
restoring torque per unit twist. The deflection φ is 90°
indicated on the scale by a pointer attached to the spring.
We have 14. The current sensitivity and voltage sensitivity of the
galvanometer depend on the number of turns of coil,
 NAB 
φ=  I magnetic field B, area A of the coil and torsion constant
 k  k of the spring or suspension wire.
5. (d) Uniform field is made radial by cutting pole pieces 15. The purpose of concave pole pieces in a moving coil
radially. galvanometer is to ensure that the plane of the coil is
6. Ratio of the magnetic moments, always parallel to the magnetic field, such that θ = 90°
1 and τ = NIBA, where symbols have their usual meanings.
πNiR 2
M′ 2 1 16. Due to eddy currents produced in core which opposes
= =
M πNiR 2 2 the cause (deflection of coil), that produces it. This
7. We can see that magnetic field is perpendicular to paper further helps in stopping the coil so on, i.e. in making
and current in the loop is in clockwise direction. So,by the galvanometer dead beat.
Fleming’s left hand rule, force on each element of the 17. Voltmeter and resistance being very high when
loop is radially outwards, so loop will have a tendency connected in series, makes the effective resistance of the
to expand. circuit very high. Due to this, current in the circuit
8. Torque, τ = NBIA sin θ, where the terms have their usual becomes extremely small.
meanings. 18. Force per unit length is
µ 2I 2
9. The principle of moving coil galvanometer is based on F = 0 [Q I1 = I 2 = I ]
the fact that when a current carrying coil is placed in a 4 πr (1/2)
magnetic field, it experiences a torque. I
If r is increased to 2r and I is reduced to , then new
10. Low torsional constant facilitates greater deflection θ 2
in coil for given value of current and hence, sensitivity µ 0 2 ( I / 2) 2
force per unit length is F ′ = ×
of galvanometer increases. 4π 2r
11. In order to produce electromagnetic damping, i.e. by 1  µ 0 2I 2 
=  ⋅ 
producing eddy currents in conducting frame which 8 4π r 
helps in stopping the coil soon.
F
12. A moving coil galvanometer can be converted into an ⇒ F ′=
8
ammeter by using a low resistance wire in parallel
F
with it. ∴ Force per unit length between them is .
8 (1½)
Moving Charges and Magnetism | 203

19. Here, first we have to find the direction of magnetic field at 23. (i) Magnetic field at centre due to circular current
point O 2 due to the wire carrying current I 1 . Use Maxwell’s µ NI
carrying coil, B = 0
right hand grip (cork screw) rule, the direction of magnetic 2r (1)
field at point O 2 due to current I 1 is along Y -axis. (ii) Magnetic moment, M = NIA = NI ( πr 2 )
Here, the wire at point O 2 is placed along Y -axis. Now,
M = πNIr 2 (1)
by the formula, F = I 2 ( l × B )
Angle between l and B is 0°, both are at Y-axis, i.e. where, r is the radius of circular coil, µ 0 is permeability
of free space and N is number of turns.
F = IlB sin 0° = 0
24. Refer to text on pages 195 and 196 (Moving coil
So, the force exerted at point O 2 because of wire along
galvanometer). (1)
X-axis is zero. (2)
Increasing the current sensitivity may not necessarily
20. (i) Refer to text on page 192 (Force between two parallel increase the voltage sensitivity, because the current
current carrying conductors). (1)
sensitivity increases with the increase of number of
(ii) ⊗B1 B2⊗ b turns of the coil but the resistance of coil also increases
a
Ia which affect adversely on voltage sensitivity. (1)
Ib 25. Refer to text on page 197 (Conversion of galvanometer
into voltmeter).
Q P 26. Refer to text on pages 195 and 196 (Moving coil
F1 galvanometer).
F2
27. We know that magnetic moment of the coil M = NIA .
Now, let the direction of current in conductor b be Since, the same wire is used in three cases with same
reversed. The magnetic field B 2 at point P due to current potentials, therefore, same current flows in three cases.
I a flowing through a will be downwards. Similarly, the
magnetic field B1 at point Q due to current I b passing Key Point: The different shapes form figures of
through b will also be downward as shown. The froce on different area and hence, their magnetic moments
a will be therefore towards the left. Also, the force on b vary.
will be towards the right. Hence, the two conductors
will repel each other as shown. (1) (i) For an equilateral triangle of side a,
N = 4, as the total wire of length = 12a
21. First, we have to find force between the wires by
assuming that one of the wires is of finite length. Then, Magnetic moment of the coil,
force is divided by length to find force per unit length.
(i ) Let length of the wire Y be l.
µ I
Force on Y , FY = 0 1 ( I 2l ), towards X.
2π a
I1 N=4
X
 3 2
M = NIA = 4 I  a 
a  4 
I2 ⇒ M = I a2 3 (1)
Y
(ii) For a square of side a, A = a 2
FY µ 0 I 1 I 2 N = 3, as the total wire of length = 12a
Force per unit length, = Magnetic moment of the coil,
l 2π a (1)
(ii) One ampere is that value of steady current which on
flowing in each of two parallel infinitely long
conductors of negligible cross-section placed in
vacuum at a distance of 1m from each other, N=3
produces a force of 2 × 10−7 newton per metre M = NIA = 3I (a 2 ) = 3I a 2 (1)
between them. (1)
(iii) For a regular hexagon of sides a,
22. Equivalent magnetic moment of the coil,
N = 2, as the total wire of length = 12a
M = IAn$ Magnetic moment of the coils,
∴ M = Ilb n$
6 3 2 N=2
where, n$ = unit vector ⊥ to the plane of the coil. M = NIA = 2 I  a  = 3 3a 2 I
∴ Torque = M × B = Ilb( n$ × B ) =0 (2)  4 
As n$ and B are parallel or anti-parallel to each other. ∴ M is in a geometric series. (1)
204 |

28. Refer to text on pages 192 and 193 (Force between two Let B = strength of the magnetic field in which coil is
parallel current carrying conductors). suspended
The direction of field at P is perpendicular to the plane and I = current passing through the coil in the direction
of the paper and outwards. The direction of field at Q is of PQRS. (1)
perpendicularly inwards to the plane of paper. Let at any instant of time, α be the angle which the
29. As the magnetic needle is placed in a two mutually normal drawn to the plane of the coil makes with the
perpendicular fields, both the fields will try to rotate the direction of magnetic field.
magnet in opposite directions. The rectangular current carrying coil when placed in the
(i ) We know that, torque on magnetic field experiences a torque whose magnitude is
BH
a magnetic dipole placed given by
in a uniform magnetic
τ = NIBA sin α
field B is
τ= M×B M Due to this torque, the coil rotates and suspension wire
gets twisted. A restoring torque is set up in the
where, M is magnetic θ
N
dipole moment and B is suspension wire.
B
magnetic field. Let θ be the twist produced in the phosphor bronze strip
The diagram (below) due to rotation of the coil and k be the restoring torque
shows the situation S per unit twist of the phosphor bronze strip.
given in question. Then, total restoring torque produced = k θ.
Torque due to B H is anti-clockwise and due to B F is In equilibrium position of the coil,
clockwise. (2)
Deflection torque = Restoring torque
(ii ) In equilibrium, | τ B H | = | τ B F |
∴ NIBA = k θ [Qα = 90°]
⇒ MB H sin θ = MB F sin (90 − θ ) k
B or I = θ = Gθ
⇒ tan θ = F NBA
BH k
where, =G
B  NBA
⇒ θ = tan −1  F 
 BH  It is known as galvanometer constant.
Here, M represents the magnetic moment. (1) ⇒ I ∝θ
30. The working of a moving coil galvanometer is based on ∴ Current is directly proportional to the deflection
the fact that when a current carrying coil is placed in a produced in the coil. (2)
magnetic field, it experiences a net torque. 31. Refer to text on page 195 (Magnetic dipole moment of a
Consider the diagram, where the coil PQRS 1 , is revolving electron). (2)
suspended freely in the magnetic field. In vector form, µ =
−e L
H 2me
Here, negative sign indicates that µ directs away from L. (1)
Phosphor
bronze strip 32. (i) Refer to text on pages 192 and 193 (Force between two
parallel current carrying conductors).
T1 T2 (ii) Refer to text on page 195 (Magnetic dipole moment of
M
a revolving electron).
P S1 33. (i) The galvanometer cannot be used to measure the
current because
Coil
(a) all the currents to be measured passes through coil
N L S
and it gets damaged easily.
(b) its coil has considerable resistance because of
length and it may affect original current. (1)
Current sensitivity of galvanometer depends on
Core Q S′ R
(a) the magnetic field
(b) the value of torsional constant. (1)
(ii) It is necessary to introduce a cylindrical soft iron core
Let l = length PQ or RS of the coil, inside the coil of a galvanometer because magnetic
field is increased, so its sensitivity increases and
b = breadth QR or SP of the coil magnetic field becomes radial. So, angle between the
and N = number of turns in the coil. plane of coil and magnetic line of force is zero in all
Area of each turn of the coil, A = l × b orientations of coil. (1)
Moving Charges and Magnetism | 205

34. According to the principle of working of a moving coil (b) an ammeter of range I , by connecting a very low
galvanometer, when a current carrying coil is placed in resistance (shunt) in parallel with galvanometer
a magnetic field, it experiences a torque. (1)
whose value is given by
I G
A high resistance that is connected in series with the S= g
galvanometer to convert into voltmeter. The value of the I − Ig (1)
resistance is given by (ii) Refer to text on pages 192 and 193 (Force between
R=
V
−G parallel current carrying conductors). (2)
Ig Thus, the nature of force is attractive.
where, V = potential difference across the terminals of the When direction of flow of current is in
voltmeter, I g = current through the galvanometer and opposite direction, the nature of force becomes
G = resistance of the galvanometer. repulsive. (1)
When resistance R1 is connected in series with the 37. Here, I = 8 A, θ = 30°, B = 0.15 T, F = ?, l = 1 m
galvanometer, then
We know that, F = BIl sin θ (1)
V
R1 = −G . ..(i) F
Ig = BI sin θ
l
When resistance R2 is connected in series with the F
galvanometer, then = 0.15 × 8 × sin30° = 0.15 × 8 × (1/2)
V l
R2 = −G ...(ii) = 0.6 N m −1 (1)
2Ig
38. Mass of wire PQ , m = 2.5 g = 2.5 × 10−3 kg
From Eqs. (i) and (ii), we get
V
= R1 − R2 and G = R1 − 2 R2
2 Ig I
P Q
The resistance R3 required to convert the given galvanometer h
into voltmeter of range 0 to 2 V is given by
2V A I C
R3 = −G
Ig
Length of wire PQ , l = 1 m
⇒ R 3 = 4 ( R1 − R2 ) − ( R1 − 2 R2 ) Current in wire PQ and AC, I = 25 A
= 3 R1 − 2 R2
Let the wire PQ rises upto a height h.
G in terms of R1 and R2 is given by G = R1 − 2 R2 (2)
The magnetic field on wire PQ due to wire AC is B.
35. For principle and working of galvanometer,
By using the formula of magnetic field due to an infinite
Refer to text on pages 195 and 196 (Moving coil length of wire,
galvanometer). (1)
µ 2I µ 2 × 25 10− 7 × 50 50 × 10− 7
(i) Cylindrical soft iron core which not only makes B= 0⋅ = 0 × = = …(i)
the field radial but also increases the strength of the 4π r 4π h h h (1)
magnet. The direction of magnetic field B on wire PQ is
(ii) Radial magnetic field is a field in which coil of perpendicularly inwards to the plane of paper (by using
the galvanometer always remains parallel to the Maxwell’s right hand rule).
field even on large deflection. (1)
Force on wire PQ, F = I ( l × B )
(iii) and (iv) refer to text on pages 195 and 196. (1½) [Q angle between l and B is 90°]
Current sensitivity does not depend upon resistance 50 × 10−7
( R ), whereas voltage sensitivity does, as evident ⇒ F = IlB sin 90°= 25 × 1 × × 1 [From Eq.(i)]
from their expression. Current sensitivity can be h
increased by increasing the number of turns of the 1250 × 10−7
coil. However, this increases the resistance of the ⇒ F = …(ii)
h (1)
coil, since voltage sensitivity decreases with
increase in the resistance of the coil the effect of The wire will lift, if the weight of the wire is balanced
increase in number of turns is nullified in the case by force due to wire AC.
of voltage sensitivity. (1½) i.e. F = mg
−7
36. (i) A galvanometer of range Ig and resistance G can be 1250 × 10
⇒ = 2.5 × 10− 3 × 9.8 [from Eq. (ii)]
converted into h
(a) a voltmeter of range V , by connecting a high
resistance R in series with galvanometer whose value 1250 × 10− 7
∴ h= = 51.02 × 10− 4 m
is given by 2.5 × 9.8 × 10− 3
V
R= −G = 51.02 × 10− 2 cm = 0.51 cm
Ig (1) Thus, the wire PQ will rise upto a height of 0.51 cm. (1)
206 |

39. Here, the angle between the magnetic field and the OE = 6 cm, OD = 10 cm, DE = EC = x
direction of flow of current is 90°. Because the magnetic D
field due to a solenoid is along the axis of the solenoid cm
and the wire is placed perpendicular to the axis. 10
6 cm x
I B O E
I x

B C
2 2 2
In ∆ODE , OD = OE + DE
Given, l = 3 cm = 3 × 10−2 m ⇒ 100 = 36 + DE 2
I = 10 A, B = 0 . 27 T ⇒ DE 2 = 64 or DE = 8 cm
The magnitude of magnetic force on the wire, and l ′ = CD = 2DE = 16 cm = 0 . 16 m
F = IlB sin90° Magnitude of force,
= 10 × 3 × 10−2 × 0.27 × sin90° F′ = I ( l × B ) = 7 ( 0.16 × 1.5 × sin 90° ) = 1.68 N
= 8.1 × 10−2 N (2)
According to Fleming’s left hand rule, the direction of
force is vertically downwards to the plane of the
According to right hand palm rule, the direction of paper. (1)
magnetic force is perpendicular to plane of paper
inwards. (1)
41. For solenoid
Given, length l = 60 cm,
40. (i) Uniform magnetic field, B = 1.5 T
Radius = 4 cm
North
Number of layers =3
West 7A Number of turns in each layer = 300
B East
6A B

South
Radius, r = 10.0 cm = 0.1 m I A
C Iw D
Current in the wire, I = 7.0 A
The magnitude of force on the wire, For wire
F = I ( l × B ) = IlB sin 90 ° Given, length, lw = 2 cm
[Q angle between l and B is 90° and the length of wire Mass, m = 2.5 g, current I w = 6 A
is equal to the diameter of the cylindrical region] Let I be the current passing through the solenoid, so the
∴ Force on the wire, magnetic field due to the solenoid.
F = I × 2r × B  Number of turns 300 × 3
B = µ 0nI Qn = = 
= 7 × 2 × 0.1 × 1.5 = 2.1 N  length 0.6 
According to Fleming’s left hand rule, the direction of 300 × 3
force is vertically inwards to the plane of paper. (1) = 4 π × 10− 7 × ×I …(i)
(ii) Now, we take the 06
. (1)
D
component of length of Force on the wire,
wire. The horizontal F = I w ( l w × B ) = I w ( lw B sin θ )
component experiences no B y
[Q angle between lw and B is 90°]
force as B is parallel to
This force balances by the weight of wire = mg
length. C x
The vertical component, ∴ I w lw B sin 90° = mg (1)
y = Diameter of the cylinder 4 π × 10− 7 × 300 × 3
So, force F = IlB sin 90° 6 × 002
. × I = 2.5 × 10− 3 × 9.8
06.
= 7 × 0.1 × 1.5 × 2 × 1 [from Eq. (i)]
= 2.1 N −3
2.5 × 10 × 9.8 × 0.6
According to the Fleming’s left hand rule, the Current, I = = 108.36 A
direction of force is perpendicularly inwards to the 108 × 4 π × 10− 7 (1)
plane of paper. (1) 42. 8 × 10−5 N ; refer to example 4 on page 193.
(iii) Let the wire be shifted by 6 cm and the position of
43. 2 × 10−5 N; refer to example 3 on page 193.
wire is CD.
Moving Charges and Magnetism | 207

44. Force per unit length between the current carrying (ii) The answer would not change as area enclosed by the
wires is given as coil as well as all other particulars remain unaltered
µ 2I I and the formula, τ = NAIB sin θ is true for planar coil
F= 0⋅ 1 2
4π r (1) for any shape. (1)
where, I 1 = current in wire AB = 12 A 49. Given, number of turns, N = 20
I 2 = current in wire CD = 5 A
Radius of circular coil, r = 10 cm = 0.1 m
and r = distance between wires = 1 mm = 1 × 10 −3 m
Magnitude of magnetic field,
µ 0 2I 1 I 2
∴ ⋅ = mg B = 0.1 T
4π r (1)
The angle between the area vector and magnetic field
where, m = mass per unit length. is 0°.
2 × 12 × 5 ⇒ θ = 0°
⇒ 10−7 × = m ×10
1 ×10−3 Current in the coil, I = 5.0 A
2 × 12 × 5 1 (i) Torque on the coil, τ = NIAB sin θ
⇒ m = 10−7 × ×
1 × 10−3 10 = 20 × 5 × π (0.1) 2 × 0.1 × sin 0° = 0
m = 1.2 × 10−3 kg/m (1)
[Qsin 0° = 0] (1)
Current in CD should be in opposite direction to that in (ii) The forces on the planar loop are in pairs, i.e. the forces
AB. on two opposite sides are equal and opposite to each
other and on the other two opposite sides, they are
45. Refer to example 5 on page 194. [Ans. 3.927 N-m]
same. Thus, the total force on the coil is zero.
46. The length of wire will be same in two cases as the same F3
coil is unwound and rewound.
Length of the wire is same.
 R
∴ N 1 × ( 2π R ) = N 2 × 2π   F1
 2 F2

[where, N 1 and N 2 = number of turns in two coils]


N 2 = 2N 1 (1)
Now, the ratio of magnetic moments is given by F4
M1 N IA N × πR12 (Q F1 = − F2 and F3 = − F4 )
= 1 1 = 1 (1)
M 2 N 2 IA 2 N 2 × πR 22 (iii) Number density of electrons, N = 1029 /m 3
2
M1  N 1   R  1 Area of cross-section of copper wire, A = 10− 5 m 2
=  ×  = ×4 = 2
M 2  2N 1   R / 2 2 The magnitude of magnetic force, F = e ( v d × B )
M 1 : M 2 = 1: 2 (1) Q I = neAv d
47. Given, N = 20, I = 12A, B = 080
. T, I
∴ vd =
neA
l = 10 cm = 10 × 10−2 m , θ = 30°
I
Q Area, A = l 2 = (10 × 10−2 )2 = 100 × 10−4 m 2 ⇒ F =e⋅ ⋅ B sin 90°
(1) NeA
As, τ = NBIA sin θ 0.1 × 5
= −5 N [Q sin90° = 1]
⇒ τ = 20 × 0.80 × 12 × 100 × 10−4 × sin 30° 10 × 1029
= 9600 × 10−4 = 5 × 10− 25 (1)
= 0.96N- m (1) 50. Here, v = 2.18 × 106 m /s , r = 5.3 ×10−11 m
48. Here, N = 30, R = 8.0cm = 8 × 10−2 m, e = 1.6 × 10−19 C
I = 6.0 A , θ = 60 ° and B = 1.0 T (i) Time period of revolution of electron is given by
(i) The magnitude of the counter torque 2πr
= magnitude of the deflecting torque T =
v
= NAIB sin θ 2π × 5.3 ×10−11
= N ⋅ ( πR 2 ) IB sin θ =
2.18 × 106
= 30 × 3.14 × (8 × 10− 2 )2 × 6.0 × 1.0 × sin 60° = 1.528 × 10−16 s
= 3.14 N - m (2)
208 |

Charge e 54. Here, n = 30, G = 100 Ω, E = 3V, R = 200 Ω, k = ?


Equivalent current, I = =
Time T Total resistance = G + R = 100 + 200 = 300 Ω
1.6 × 10−19 E 3 1
I= Ig = = = A
1.528 × 10−16 G + R 300 100 (1)
⇒ I = 1.05 ×10−3 A (1) I g 1 /100
k= =
(ii) Field at proton due to orbiting electron is n 30
µ I µ 2π I 1 −3
B = 0 or B = 0 ⋅ = × 10 A /div
2r 4π r 3
10 × 2π × 1.05 × 10−3
−7 1
B= k = × 10− 3 × 106 µA/ div
5.3 × 10−11 3
(1)
Figure of merit (Restoring torque per unit twist)
= 12.4 T
k = 3333 . µA /div (1)
51. Here, B = 60 G, A = 2 × 10− 4 m2 , N = 40
55. Given, resistance of galvanometer coil, G = 12 Ω
I = 4 mA = 4 × 10−3 A, θ = 16° Current in galvanometer,
k
Q I = θ R
NBA G
NBAI
⇒ k= Ig
θ (1)
40 × 60 × 2 × 10− 4 × 4 × 10−3 V
=
16 I g = 3 mA = 3 × 10− 3 A
= 1. 2 × 10− 4 N-m per degree (1)
and potential difference,
52. Given, R1 = 10 Ω, N 1 = 30, A1 = 3.6 × 10− 3 m2 , V = 18 V
B1 = 0 . 25 T, R2 = 14 Ω, N 2 = 42,
We can convert the galvanometer into voltmeter by
A2 = 1.8 × 10− 3 m 2 , B2 = 0 .50 T
using a large resistance R in series. The resistance can
k1 = k 2 (spring constants are same) ...(i) be calculated using the formula,
(i) Using the formula of current sensitivity, V
NAB R= −G
I = Ig
k
IS 2 N 2 B 2 A 2k1 42 × 0.50 × 1.8 × 10− 3 18
∴ = = R= − 12
IS 1 N 1 B1 A1k 2 30 × 0.25 × 3.6 × 10− 3 3 × 10− 3
= 1.4 [From Eq. (i)] (1½) = 5988 Ω (1)
(ii) Using the formula of voltage sensitivity, This resistance (R = 5988 Ω) is connected in series with
NAB the galvanometer. The resistance is connected in series
V = because we have to increase the resistance of the
kR
VS 2 N 2 B 2 A 2k1 R1 galvanometer, so that almost no current flows through it
∴ = and it gives an exact value of potential difference.
VS 1 k 2 R 2 N 1 B1 A1
I – Ig
42 × 0.50 × 1.8 × 10− 3 × 10 G
=
14 × 30 × 0.25 × 3.6 × 10− 3 Ig
=1 [from Eq. (i)] (1½)
53. Refer to Example 10 on page 197. [Ans. 001. Ω] S (1)
SUMMARY
¢
Magnetic Field The space in the surroundings of a magnet or velocity (v ) inside a uniform magnetic field, then force acting on
a current carrying conductor in which its magnetic influence it is given by F = q(v × B ) .
can be experienced is called magnetic field. ¢ Magnetic Force On a Charged Particle It is given by
¢ Oersted’s Experiment HC Oersted by his experiment F = q (v × B ) .
observed that a current carrying conductor deflects magnetic ¢ When charged particle enters into a magnetic field
compass needle placed near it. perpendicularly, then
¢ Biot-Savart’s Law This law deals with the magnetic field mv 2 mv 2πm
(i) = qvB (ii) r = (iii) T =
induction at a point due to a small current element, i.e. r qB qB
Idlsin θ qB q 2 B2 r 2
dB ∝ (iv) ν = (v) KE =
r2 2 πm 2m
¢ Permittivity and Permeability Electric permittivity (ε 0 ), the ¢ Lorentz Force The sum of the electric force and magnetic
degree of interaction of electric field with medium. force that can be exerted on a charged particle due to its
Magnetic permeability, the ability of a substance to acquire electric charge (q) is called Lorentz force.
magnetisation in a magnetic field. It is F = q (E + v × B )
¢ Right Hand Thumb Rule When the thumb of right hand is ¢ Cyclotron It is used to accelerate the charged particles or
placed along the direction of current, the fingers curl around ions to high energy. Electric and magnetic fields are used in
the conductor in the direction of magnetic field lines. combination to increase the energy.
¢ Magnetic field at any point along the axis of circular current ¢ Force on a Current Carrying Conductor in a Uniform
µ 0 Ia2 Magnetic Field It is given by, F = IlB sin θ
carrying conductor is B =
2 (r + a2 )3 / 2
2
¢ Fleming’s Left Hand Rule If the forefinger, middle finger and
¢ Magnetic field at the centre of a circular current carrying the thumb of the left hand are stretched mutually at right
conductor/ coil angles to one another such that the forefinger points in the
µ I direction of magnetic field, middle finger in the direction of
B= 0
2r current, then thumb will point in the direction of force on the
conductor.
¢ Ampere’s Circuital Law According to this law, the line integral
of the magnetic field B around any closed path in vacuum is
¢
Force between Two Parallel Current Carrying Conductors
 µ 2 I1I 2 
equal to µ 0 times the net current enclosed by the curve, It is given by F= 0
i.e.  4π r 
∫ B × d l = µ 0I
¢ Torque Experienced by a Current Loop in a Uniform
¢ Magnitude of a Magnetic Field of a Straight Wire It is given Magnetic Field It is given by τ = B I NA sin θ
µ I
by B= 0 ¢ Circular Current Loop as Magnetic Dipole The magnitude of
2 πr
the magnetic field on the axis at a distance x from the centre of
¢ Solenoid It is an insulated long wire closely wound in the form a circular loop of radius R carrying a steady current I is
of a helix. µ 0 IR 2
B=
¢ Magnetic Field due to Straight Solenoid At any point inside 2 ( x 2 + R 2 )3 / 2
the solenoid, B = µ 0 nI
For x >> R
At points near the end of air closed solenoid, B = (µ 0 nI / 2 ) .
µ 0 IA µ M
¢ Toroid An endless solenoid in the form of a ring is called a B= 3
or B = 0 3
2x 2π x
toroid. Its magnetic field is B = µ 0 nI.
¢ Force on Moving Charge in a Uniform Magnetic
¢ Moving Coil Galvanometer It is an instrument which is based
Field When a charged particle (q ) moves with a on the fact that when a current carrying coil is placed in a
magnetic field, then it experiences a torque.

For Mind Map


Visit : https://goo.gl/m1mM2z OR Scan the Code
Chapter Practice
8 State the rule that is used to find the
Multiple Choice Questions direction of magnetic field at a point near a
current carrying straight conductor.
1 Mark
9 What will be the magnetic field at the centre
1 A long solenoid has 20 turns cm −1 . The current of a circular coil carrying current, when the
necessary to produce a magnetic field of 20 mT current through the coil is doubled and the
inside the solenoid is approximately radius of the coil is halved?
(a) 1A (b) 2 A
(c) 4 A (d) 8 A 10 What is the force on a charge moving along
the direction of the magnetic field?
2 For a toroid, magnetic field strength in the region
enclosed by wire turns is given by 11 Name the force which is experienced by a
moving charged particle in electric and
(a) B = µ 0n I, where n = number of turns
magnetic field.
(b) B = µ 0 I /n , where n = number of turns per metre
µ 0I
12 Under what condition does an electron moving
(c) B = , where r = mean radius through a magnetic field experience maximum
2r
force?
µ 0 NI where N = number of turns
(d) B = , 13 A charged particle moves through a magnetic
2 πr  and r = radius of toroid
field. Is the momentum of the particle
3 Magnetic field produced by a wire carrying affected?
current
(a) is parallel along the axis of wire 14 An electron beam projected along + X-axis
(b) is perpendicular to the plane of the wire experiences a force due to a magnetic field
(c) forms circular loops along the axis of wire and along the + Y -axis. What is the direction of
coplanar to the wire the magnetic field?
(d) direction of field is not constant
15 In a certain arrangement, a proton does not
4 A cyclotron is used to accelerate charged particles
get deflected while passing through a
or ions to high energies. It uses
magnetic field region. Under what condition
(a) only electric field
is it possible?
(b) only magnetic field
(c) Both electric and magnetic fields 16 Write the expression for the force between
(d) None of the above parallel current carrying conductors.
5 Current sensitivity of a galvanometer is
(a)
NBA
(b)
k
(c)
NBA
(d)
kR Short Answer Type I Questions
k NBA kR NBA 2 Marks

Very Short Answer Type Questions 17 Two similar coils are placed mutually
perpendicular such that their centres
1 Mark coincide. At centre, what will be the ratio of
6 What is the source of magnetic field produced due the magnitudes of magnetic fields due to one
to flow of current in a wire? coil and the resultant magnetic field?

7 What is the value of magnetic field induction at 18 In what way, current carrying solenoid
any point on the thin conducting wire carrying behaves like a bar magnet. Find the magnetic
current? field induction at the axis of solenoid due to
current flowing through it.
Moving Charges and Magnetism | 211

19 What is Lorentz force? Give some important Numerical Problems


characteristics of this force. 29 A copper coil of 100 turns, radius 8 × 10 −2 m
20 Equal currents are flowing through two carries a current of 0.40 A. What will be the
infinitely long parallel wires in the same magnitude of magnetic field at the centre
direction. What will be the magnetic field at a of coil? (2 M)
point mid-way between the two wires? 30 A straight wire carrying a current of 12 A is bent
21 Deduce an expression for the torque on a into a semi-circular arc of radius 2.0 cm as
current carrying loop suspended in a uniform shown in Fig. (a). Consider the magnetic field B
magnetic field. at the centre of the arc.

22 In a moving coil galvanometer having a coil of


N turns of area A and carrying current I and is
placed in a radial field of strength B. What will (a)
be the torque acting on the coil?
23 Is it possible to decrease or increase the range
of given voltmeter? Explain. (b)

(i) What is the magnetic field due to the


Short Answer Type II Questions straight segments?
3 Marks (ii) In what way the contribution to B from the
semi-circle differs from that of a circular
24 Using Ampere’s circuital law, find an expression loop and in what way does it resemble?
for the magnetic field at a point situated at a (iii) Would your answer be different, if the wire
normal distance R from an infinitely long was bent into a semi-circular arc of the
current carrying straight wire. same radius but in the opposite way as
25 An electron being accelerated through a shown in Fig. (b)? (3 M)
potential difference of V enters a uniform 31 A closely wound solenoid 0.80 m long has 5
magnetic field of B perpendicular to the layers of windings of 400 turns each. The
direction of motion. Find the radius of path diameter of the solenoid is 1.8 × 10 −2m. If the
described by the electron. current carried is 0 .8 A, what will be the
26 A wire of length L is bent in the form of a circle magnitude of field near the centre? (3 M)
of radius R and carries current I. What is its 32 A toroid of core of inner radius 0.25 m and
magnetic moment? outer radius 0.26 m around which 3500 turns of
a wire are wound. If the current in the wire is 11
Long Answer Type Questions A. What will be the magnetic field inside the
5 Marks core of toroid? (3 M)

33 A beam of protons passes undeflected with a


27 Three wires of equal lengths are bent into the
horizontal velocity v, through a region of
form of three loops. One of the loops is
square-shaped, second loop is electric and magnetic fields, mutually
triangular-shaped and third loop is circular. perpendicular to each other and normal to the
These are suspended in a uniform magnetic direction of beam.
field and the same current is passed through If the magnitudes of electric and magnetic
them. Which loop will experience greater fields are 100 kV/m and 50 mT respectively,
torque? Give reasons. calculate the velocity of the beam. (2 M)
28 A rectangular current
A
I
B
34 Deuterons in a cyclotron describe a circle of
carrying loop of length radius 32 cm just before emerging from the
a and breadth b is O dees. The frequency of the applied alternating
I I b voltage is 10 MHz. Find
shown in the figure.
Find the magnetic field (i) the flux density of magnetic field and
at the centre of the D C
I (ii) the energy and the speed of the deuterons
loop. a
upon emergence. (3 M)
212 |

35 Two concentric circular wire loops of radii 20 3 (d) The Oersted found that the I
cm and 30 cm are located in an XY -plane, each alignment of the magnetic needle is
carries a clockwise current of 7 A. tangential to an imaginary circle
which has the straight B
(i) Find the magnitude of the net magnetic current-carrying wire, as its centre
dipole moment of the system. has its plane perpendicular to the
(ii) Repeat for reversed current in the inner wire as shown in figure.
loop. (5 M) 4 (c) The cyclotron is a machine to accelerate charged
particles or ions to high energies. The cyclotron used
36 The coil of galvanometer consists of 100 turns both electric and magnetic fields in combination to
and effective area of 1 cm 2. The restoring couple increase the energy of charged particles.
is 10 − 8 N-m/rad. The magnetic field between 5 (a) Current sensitivity of the galvanometer deflection
poles is of 5 T. What will be the current per unit ampere
sensitivity of galvanometer? (3 M) φ NBA
⇒ =
I k
37 The current sensitivity of a MCG increases by
6 Flow of current refers to moving electrons which are the
20% when its resistance is increased by a factor
source of magnetic field.
of 2. Calculate by what factor the voltage
7 As given that the angle between the length of
sensitivity changes? (3 M)
conducting wire and at the point at which magnetic
38 A galvanometer with a coil of resistance 12.0 Ω field is to be determined is zero.
shows full scale deflection for a current of 2.5 ∴ B=0
mA. How will you convert this meter into 8 Right hand thumb rule states that, if we imagine a linear
(i) an ammeter of range 0 to 7.5 A? wire conductor to be need in the grip of the right hand
(ii) a voltmeter of range 0 to 10 V? Determine such that the thumb points in the direction of current,
the net resistance of the meter in each then the curvature of the fingers around the conductor
will give the direction of magnetic field lines.
case. When an ammeter is put in a circuit,
does it read less or more than the actual 9 Magnetic field at the centre of the coil is given by
µ I
current in the original circuit? When a B= 0
voltmeter is put across a part of the circuit, 2R
does it read less or more than the required µ 0 ( 2I )
B′ = = 4B
voltage drop? Explain. (5 M) 2( R / 2)
10 Force on a moving charge in magnetic field is given as,
39 A galvanometer having 30 divisions has a
F = qvB sinθ
current sensitivity of 20 µA/ div. It has a
resistance of 25 Ω. Here, θ = 0° ⇒ F = 0
11 Lorentz force
(i) How will you convert it into an ammeter of
12 Magnetic forec, F = q( v × B ) = qvB sinθ
range 0-1 A?
Maximum force, F max = qvB
(ii) How will you convert this ammeter into a
voltmeter of range 0-1 V? (3 M) When, sinθ = 1 or θ = 90°
13 No, its momentum does not get affected.
14 Direction of magnetic field is in Z-axis direction.
15 When it is along the magnetic field.
Answers 16 Force between the parallel current carrying conductors
µ 2I I
1 (d) By the formula, is F = 0 1 2 .
4π r
B = µ 0nI or 20 × 10−3 = 4 π × 10−7 × 2000 × I B1 1
17 Ratio = =
20 × 10−3
2 2
B1 + B 2 2
or I = ~ 8A
⇒ I −
4 π × 10−7 × 2000 18 Refer to text on pages 176 and 177 (Magnetic field of
solonoid).
2 (d) For toroid, applying Ampere’s circuital law,
µ NI 19 Refer to text on page 179 (Force on a moving charge in a
B ( 2πr ) = µ 0 NI ⇒ B = 0 uniform magnetic and electric field (Lorentz force)).
2π r 20 Zero
where, B = magnetic field of a toroid, 21 Refer to text on pages 193 (Torque experienced by a
N = number of turns of toroidal coil current loop in uniform magnetic field (Magnetic
and r = radius of toroid. dipole)).
Moving Charges and Magnetism | 213

22 Refer to text on pages 195 and 196 (Moving coil (ii) Magnetic field due to a semi-circular wire at its centre
galvanometer). is half of magnetic field due to a circular loop.
µ I
23 Yes, we can increase the range of voltmeter by Bsemi -circle = 0 = 19 . × 10 −4 T
connecting additional resistance in series with the 4r
voltmeter. We can decrease the range of voltmeter by (iii) The magnitude of the magnetic field remains same
connecting a suitable resistance in parallel with the but the direction will be opposite.
voltmeter. 31 Refer to example 2 on page 177.
24 Refer to text on page 176 (Magnitude of magnetic field N
[Ans. B = µ 0 I = 2.5 × 10−2 T ]
of a infinite long straight wire using Ampere’s circuital l
law). 32 Refer to example 4 on page 178.
1
25 K = mv 2 = eV , r =
mv mv 2mV [Ans. B toroid = µ 0nI = 3 × 10−2 T ]
= =
2 qB eB eB 2 33 For undeflected beam,
L E
26 As, L = 2πR ⇒ R = v = = 2 × 106 m/s
2π B
⇒ M = IA = I × πR 2 34 Refer to example 6 on page 179.
L2 I L2 (i) B =1.31 Wb/m 2
= Iπ× 2 =
4π 4π 1
(ii) K = mv 2 = 4.21 MeV
27 Refer text on pages 193 and 194 (Torque experienced by 2
a current loop in uniform magnetic field (Magnetic 35 (i) M1 = N 1 I 1 A1 ⊗
dipole)). M 2 = N 2I 2A2 ⊗ I
Now, apply the formula, τ = MB . A-m 2
∴ M = M1 + M 2 = 286 20 cm
Square will experience maximum torque. 2
(ii) M = | M1 − M 2 | = 110
. A-m 30 cm
 2µ 0 I a 2 + b 2  NAB
28 Refer to example 2 on page 165. Ans. B =  36 Current sensitivity, I s =
 πab  K
100 × 1 × 10−4 × 5 6 −1
29 Refer to example 4 on page 167. = = 5 × 10 A
N µ 0I 10−8
[Ans. B = . × 10−4 T]
= 31 37 Refer to example 9 on page 196.
2r
[Ans. Decreased by a factor 0.4.
30 (i) The magnitude of the magnetic field at a point distant 38 Refer to example 10 on page 197.
r from a current element δl is [Ans. (i) Resistance of ammeter 4 × 10−3 Ω
µ i δl sin θ
B= 0⋅ (ii) Resistance of voltmeter = 4000 Ω
4π r2
39 Refer to example 10 and 11 on page 197.
Here, for each straight part of the wire, θ = 0°.
Ans. (i) Shunt = 0015
. Ω
Hence, for each straight part of length l, we have
(ii) Resistance in series 0985
. Ω
B1 = B 2 = 0

RELATED ONLINE VIDEOS


Visit : https://www.youtube.com/watch? Visit : https://www.youtube.com/watch?
v=1BoIH6Quhiw v=m2jp0klZHEE
OR Scan the Code OR Scan the Code

Visit : https://www.youtube.com/watch? Visit : https://www.youtube.com/watch?


v=roJO_T0eKDM v=09HutPipRGk
OR Scan the Code OR Scan the Code

Visit : https://www.youtube.com/watch?
v=a97NpQKIbks
OR Scan the Code

Das könnte Ihnen auch gefallen